Download as pdf or txt
Download as pdf or txt
You are on page 1of 189

6/9/24, 12:45 PM Neet PG Preparation, Neet PG Coaching, FMGE, USMLE

Grand Test-9 (Clinical New Pattern)

RESULT

View View View SectionWise


Result
leaderboard Analysis Solution Analysis

Grand Test-9 (Clinical New Pattern)


Question Wise Report

Question: 1
Nerve supply of the tip of nose is :

A Infratrochlear

B Supratrochlear

C Supraorbital

D External nasal

29.89% People got this right

Explanation:

Correct Answer (D)

External nasal

Ref: Read the text below

Sol :

https://emedicoz.com/testresult/683510s14692547 1/189
6/9/24, 12:45 PM Neet PG Preparation, Neet PG Coaching, FMGE, USMLE

The tip of nose is supplied by external nasal a branch of ophthalmic division of


trigeminal nerve.

Question: 2
While removing a fish bone stuck in the throat, the main trunk of internal
laryngeal nerve was severed. The bone was stuck in the :

A Piriform fossa

B Sinus of larynx

C Vallecula epiglottis

D Vestibule of larynx

54.4% People got this right

Explanation:

Correct Answer (A)

Piriform fossa

Ref: Read the text below

Sol :

On either side of the laryngeal orifice is a recess, termed the piriform sinus (also
piriform recess, pyriform sinus, piriform fossa, or smuggler's fossa), which is
bounded medially by the aryepiglottic fold, laterally by the thyroid cartilage and
thyrohyoid membrane. The fossae are involved in speech.

Deep to the mucous membrane of the piriform fossa lie the internal laryngeal
nerve, a branch of the superior laryngeal nerve.

https://emedicoz.com/testresult/683510s14692547 2/189
6/9/24, 12:45 PM Neet PG Preparation, Neet PG Coaching, FMGE, USMLE

The internal laryngeal nerve supplies sensation to the area, and it may become
damaged if the mucous membrane is inadvertently punctured

Question: 3
Example of traction epiphysis is :

A Os trigonum of talus

B Coracoid process of
scapula

C Head of femur

D Tibial tuberosity

36.89% People got this right

Explanation:

Correct Answer (D)

Tibial tuberosity

Ref: Read the text below

Sol :

• Coracoid process & Os Trigonum of Talus are eg. of Atavastic epiphysis.

• Attached to the tip are the origins of short head of biceps & coracobrachialis.

• Attached to the medial border is the insertion of pectoralis minor.

• Greater tubercle & tibial tuberosity are examples of traction epiphysis, head
of femur is a type of Pressure epiphysis.

https://emedicoz.com/testresult/683510s14692547 3/189
6/9/24, 12:45 PM Neet PG Preparation, Neet PG Coaching, FMGE, USMLE

Question: 4
Association Fibers are white matter fibers which connect :

A Corresponding areas in
two cerebral hemisphere

B Cerebral cortex with lower


level in brain

C Different cortical areas in


the same hemisphere

D Different cranial nerve


nuclei

45.16% People got this right

Explanation:

Correct Answer (C)

Different cortical areas in the same hemisphere

Ref.: Read the text below

Sol :

Association fibers : Connect different cortical areas in the same hemisphere


Projection Fibers : Connect cerebral cortex with lower level in brain (Eg.
Brainstem, spinal cord)
Commissural fibers : Connect corresponding areas in the two cerebral
hemispheres

Question: 5
False regarding Erb’s paralysis?

https://emedicoz.com/testresult/683510s14692547 4/189
6/9/24, 12:45 PM Neet PG Preparation, Neet PG Coaching, FMGE, USMLE

A Difficulty in adduction

B Difficulty in abduction

C Difficulty in supination

D Policeman tip hand

46.39% People got this right

Explanation:

Correct Answer (A)

Difficulty in adduction

Ref: Read the text below

Sol : Adducted and medial rotated arm, flexed and pronated forearm is seen in
Erb’s paralysis

Question: 6
Left 6th arch artery gives rise to which among the following?

A Ductus arteriosus

B 1st vertebral artery

C Proximal subclavian artery

D Arch of Aorta

27.86% People got this right

https://emedicoz.com/testresult/683510s14692547 5/189
6/9/24, 12:45 PM Neet PG Preparation, Neet PG Coaching, FMGE, USMLE

Explanation:

Correct Answer (A)

Ductus arteriosus

Ref: Read the text below

Sol : Distal part of Left 6th arch artery formsDuctus arteriosus

Question: 7
A patient with which of the following eye diseases would be at greatest
risk for retinal damage from hypotension during surgery?

A Strabismus

B Open eye injury

C Glaucoma

D Severe myopia

21.42% People got this right

Explanation:

Correct Answer (C)

• Blood flow to the retina can be decreased by either a decrease in mean


arterial pressure or an increase in intraocular pressure.

• Decreased blood flow and stasis are more likely in patients with glaucoma
because of their elevated intraocular pressure.

• During periods of prolonged hypotension, the incidence of retinal artery


thrombosis increases in these patients

https://emedicoz.com/testresult/683510s14692547 6/189
6/9/24, 12:45 PM Neet PG Preparation, Neet PG Coaching, FMGE, USMLE

Question: 8
Anesthesia is induced in a 50-year-old, 125-kg man for laparoscopic
cholecystectomy. The patient is placed on a ventilator. Peak airway
pressure is noted to be 20 cm h2o with o2 saturation 99% on pulse
oximeter. An hour later, the peak airway pressure rises to 40 cm h2o and
paco2 is 38 mm hg on infrared spectrometer and on o2 Saturation falls to
88%. Blood pressure and heart rate are unchanged. The most likely cause
of these findings is

A Mainstem intubation

B Thrombotic pulmonary
embolism

C Tension pneumothorax

D Venous air embolism

17.95% People got this right

Explanation:

Correct Answer (A)

Symptoms of a mainstem or bronchial intubation include

• Asymmetric chest expansion, unilateral breath sounds, elevation of peak


airway pressures, and abg abnormalities (e.g., hypoxemia).

• Frequently, bronchial intubation is intentional (e.g., thoracic surgery with


double-lumen endotracheal tubes), but, if undetected with a single-lumen tube,
atelectasis, hypoxia, and pulmonary edema may result in time.

• Peak airway pressures can also increase with many conditions such as airway
obstruction (e.g., kinked endotracheal tube, secretions, overinflated cuffs),
bronchospasm, increase in chest wall muscle tone (rigid chest with narcotics,
coughing), and tension pneumothorax.

https://emedicoz.com/testresult/683510s14692547 7/189
6/9/24, 12:45 PM Neet PG Preparation, Neet PG Coaching, FMGE, USMLE

• If a tension pneumothorax develops, associated hypotension usually is


present and fall in etco2

• Pulmonary embolism would not cause the peak airway pressure to rise as in
this case and will cause sudden and abrupt fall in etco2

Question: 9
Which statement concerning local anesthetics is correct?

A The un-ionized form of a


local anesthetic binds to the
nerve membrane to block
conduction

B If one node of Ranvier is


blocked, conduction will be
reliably interrupted

C The presence of myelin


enhances the ability of a local
anesthetic to block nerve
conduction

D Local anesthetics block


transmission by inhibiting the
voltage-gated potassium ion
channels

24.65% People got this right

Explanation:

Correct Answer (C)

• The un-ionized form of the local anesthetic traverses the nerve membrane,
whereas the ionized form blocks conduction.

• About three nodes of Ranvier must be blocked to achieve anesthesia.

https://emedicoz.com/testresult/683510s14692547 8/189
6/9/24, 12:45 PM Neet PG Preparation, Neet PG Coaching, FMGE, USMLE

• The presence of myelin enhances the ability of a local anesthetic to block


conduction, as does rapid firing.

• The local anesthetic blocks nerve transmission by inhibiting the voltage-


gated sodium ion channels

Question: 10
Which one of the following statements about the fed and fasting
metabolic states is correct?

A Fatty acids and


triaclyglycerol are in the
synthesized in the liver in the
fasting state.

B In the fasting state the


main fuel for the central nervous
system is fatty acids released
from adipose tissue.

C In the fasting state the


main metabolic fuel for most
tissue comes from fatty acids
released from adipose tissue.

D In the fed state muscle


cannot take up glucose for use as
a metabolic fuel because glucose
transport in muscle is stimulated
in response to glucagon.

43.41% People got this right

Explanation:

Correct Answer (C)

https://emedicoz.com/testresult/683510s14692547 9/189
6/9/24, 12:45 PM Neet PG Preparation, Neet PG Coaching, FMGE, USMLE

In the fasting state the main metabolic fuel for most tissue comes from fatty
acids released from adipose tissue.

During fasting state level of glucagon is high which will activate hormone
sensitive lipase enzyme in adipose cell and will mobilize fatty acid from
triacylglycerol.

Question: 11
A 54-year-old man with type 1 (IDDM) diabetes is referred to an
ophthalmologist for evaluation of developing cataracts. Pre-appointment
blood work was requested and the results are shown below:

Which of the following enzymes is most strongly associated with cataract


formation in this patient?

A Galactokinase

B Aldose reductase

C Glucokinase

D Aldolase B

35.47% People got this right

Explanation:

https://emedicoz.com/testresult/683510s14692547 10/189
6/9/24, 12:45 PM Neet PG Preparation, Neet PG Coaching, FMGE, USMLE

Correct Answer (B)

Aldol reductase

Aldol reductase is the enzyme which is found in lens cell and is responsible for
reduction of glucose and galactose in to sorbitol and galactitol respectively.
These are hygroscopic alcohols and are responsible for cataract formation in
diabetes and galactosemia respectively

Question: 12
Proximal Histidine is

A E7

B E8

C F7

D F8

6.52% People got this right

Explanation:

Correct Answer (D)

F8

Distance of Fe++ of heme to the nitrogen of imidazole of histidine in globin


chain is taken into consideration to decide the nomenclature of histidine as
proximal or distal.

Proximal histidine: F8

Distal Histidine: E7

https://emedicoz.com/testresult/683510s14692547 11/189
6/9/24, 12:45 PM Neet PG Preparation, Neet PG Coaching, FMGE, USMLE

Question: 13
Following are the test done for protein, sugar, ketone body. Which will be
positive in urine in the state of starvation.

A 1&2

B Only 2

C Only 3

D 2&3

51.14% People got this right

Explanation:

Correct Answer (C)

Only 3

During starvation urine may have ketone bodies, which will give positive
Rothera’s test.

Test tube 3 shows positive Rothera’s test which is for ketone bodies

Test tube 1 shows positive Biuret test which is for proteinuria

Test tube 2 shows positive Benedict’s test which is for glycosuria

https://emedicoz.com/testresult/683510s14692547 12/189
6/9/24, 12:45 PM Neet PG Preparation, Neet PG Coaching, FMGE, USMLE

Question: 14
Which of the following statements about purine nucleotide metabolism is
NOT CORRECT?

A An early step in Pyrimidine


biosynthesis is the formation of
PRPP (phosphoribosyl 1-
pyrophosphate).

B Inosine monophosphate
(IMP) is a precursor of both AMP
and GMP.

C Orotic acid is an
intermediate in pyrimidine
nucleotide biosynthesis

D Ribonucleotide reductase
converts nucleoside diphosphates
to the corresponding deoxyribo
nucleoside diphosphates.

13.83% People got this right

Explanation:

Correct Answer (A)

An early step in Pyrimidine biosynthesis is the formation of PRPP


(phosphoribosyl 1-pyrophosphate).

Phosphoribosyl 1-pyrophosphate( PRPP) is starting material required in purine


nucleotide biosynthesis but in pyrimidine nucleotide biosynthesis, PRPP is
required at much later step when aorotic acid is already been formed.

PRPP is the activated form of ribose 5 phosphate which is required for synthesis
of purine as well as pyrimidine nucleotides( both in denovo and salvage route)

https://emedicoz.com/testresult/683510s14692547 13/189
6/9/24, 12:45 PM Neet PG Preparation, Neet PG Coaching, FMGE, USMLE

Question: 15
Supraglottoplasty is the treatment of choice for severe cases of:

A Subglottic stenosis

B Laryngomalacia

C Posterior laryngeal cleft

D Vocal cord paralysis

40.57% People got this right

Explanation:

Correct Answer (B)

Laryngomalacia

Ref: Read the text below

Sol:

• Severe laryngomalacia can result in GERD, laryngeal penetration and


aspiration, failure to thrive, apnea, pectus excavatum, and cyanosis.

• Supraglottoplasty (aryepiglottoplasty) is the standard treatment for severe


laryngomalacia.

• Laryngomalacia can occur in older children and be a cause of obstructive


sleep apnea, particularly in neurologically impaired children.

Question: 16
OAEs provide information about which of the following?

https://emedicoz.com/testresult/683510s14692547 14/189
6/9/24, 12:45 PM Neet PG Preparation, Neet PG Coaching, FMGE, USMLE

A Integrity of the distal


segment of the cochlear nerve

B Integrity of the proximal


segment of the cochlear nerve

C Frequency-specific regions
of the cochlea

D Inferior colliculus

26.07% People got this right

Explanation:

Correct Answer (C)

Frequency-specific regions of the cochlea

Ref: Read the text below

Sol:

OTOACOUSTIC EMISSIONS

• Audio frequency signals that reflect the motility status of the cochlea's outer
hair cells

• Outer hair cells serve as a cochlear partition of amplifiers.

• Their acoustic by-products are OAEs.

Measure OAEs by placing a microphone and loudspeaker into a sealed EAC

Question: 17
Which of the following statements are correct?

i. Griesinger sign refers to erythema and edema behind the mastoid


process resulting from septic thrombi of the mastoid emissary veins in
https://emedicoz.com/testresult/683510s14692547 15/189
6/9/24, 12:45 PM Neet PG Preparation, Neet PG Coaching, FMGE, USMLE

lateral sinus thrombosis.

ii. Delta sign refers to the appearance of a lateral sinus thrombosis on


contrast-enhanced radiographic imaging resulting from the nonenhancing
clot surrounded by enhancing dural sinus wall.

iii. The halo sign, a clinically detectable single ring sign.

iv. Brown sign refers to blanching of the tympanic membrane when


applying positive pressure and can be seen in glomus jugulare tumors.

v. Bezold sign refers to bruising behind the ear at the mastoid process
and is seen in fractures of the petrous temporal bone.

A i, ii, iii, and iv

B i, ii, and iv

C ii, iii, and iv

D i, ii, and v

32.61% People got this right

Explanation:

Correct Answer (B)

i, ii, and iv

Ref: Read the text below

Sol:

• Griesinger sign refers to erythema and edema behind the mastoid process
resulting from septic thrombi of the mastoid emissary veins in lateral sinus
thrombosis.

• Delta sign refers to the appearance of a lateral sinus thrombosis on contrast-


enhanced radiographic imaging resulting from the nonenhancing clot

https://emedicoz.com/testresult/683510s14692547 16/189
6/9/24, 12:45 PM Neet PG Preparation, Neet PG Coaching, FMGE, USMLE

surrounded by enhancing dural sinus wall.

• The halo sign, a clinically detectable double ring.

• Brown sign refers to blanching of the tympanic membrane when applying


positive pressure and can be seen in glomus jugulare tumors.

• Battle sign refers to bruising behind the ear at the mastoid process and is
seen in fractures of the petrous temporal bone.

Question: 18
A 70 year old male presents with a 3 month history of right pulsatile
tinnitus with hearing loss, unstable gait,and autophany.ECOG
demonstrates a SP/AP ratio of 0.6. A diagnosis of superior semicircular
cancal dehiscence is suspected. Which of the following is a good indicator
test that would give support to this diagnosis:

A Otoacoustic emissions

B Tympanometry

C Cervical vestibular evoked


myogenic potential

D Auditory brainstem
response

37.13% People got this right

Explanation:

Correct Answer (C)

Cervical vestibular evoked myogenic potential

Ref: Read the text below

Sol:
https://emedicoz.com/testresult/683510s14692547 17/189
6/9/24, 12:45 PM Neet PG Preparation, Neet PG Coaching, FMGE, USMLE

• Superior semicircular canal dehiscence (SSCD) usually demonstrates an


elevated SP/AP ration above 0.4 on ECOG.

• An additonal test that can be a good diagnositic indicator is cervical


vestibular evoked myogenic potential (cVEMP).

• This tests the function of the saccule and the inferior vestibular nerve.

• In SSCD the threshold of the cVEMP is abnormally low (65 dB or less) making
this test a good indicator of SSCD.

Question: 19
Tonsillitis associated with a gray membrane covering the surface of the
tonsil but that may be removed without bleeding is often associated with
which of the following?

A HPV

B EBV

C CMV

D HSV

39.71% People got this right

Explanation:

Correct Answer (B)

EBV

Ref: Read the text below

Sol:

• A gray membrane may cover tonsils that are inflamed from an EBV infection
(see the image below).
https://emedicoz.com/testresult/683510s14692547 18/189
6/9/24, 12:45 PM Neet PG Preparation, Neet PG Coaching, FMGE, USMLE

• This membrane can be removed without bleeding.

• Palatal mucosal erosions and mucosal petechiae of the hard palate may also
be observed.

Question: 20
All of the following are true about tube shown below except

A High tracheostomy may


lead to subglottic stenosis

B Anterior Jugular veins and


inferior thyroid veins are the most
common site of bleeding

C In pediatric tracheostomy,
a circular ring of anterior tracheal
wall is removed

D Percutaneous dilatational
tracheostomy is used mostly in
ICU patients

24.23% People got this right

Explanation:

https://emedicoz.com/testresult/683510s14692547 19/189
6/9/24, 12:45 PM Neet PG Preparation, Neet PG Coaching, FMGE, USMLE

Correct Answer (C)

In pediatric tracheostomy, a circular ring of anterior tracheal wall is removed

Ref: Read the text below

Sol:

• High trachestomy means when it starts from first ring. It is only done in a
case of cancer larynx. Otherwise, the level of tracheostomy is 2nd and 3rd rings.

• If we involve first ring, it can cause subglottic stenosis later on.

• In pediatric trachestomy, we don’t remove circular ring of anterior tracheal


wall because this can lead to tracheomalacia later on.

• In adults we do remove a circular ring of anterior tracheal wall to place the


tube.

Question: 21
CROS or BiCROS systems are most appropriate for patients with which of
the following?

A Symmetric hearing loss

B Significantly poorer
hearing loss in one ear

C Symmetric word
recognition scores

D Poor word recognition


scores bilaterally

18.9% People got this right

Explanation:

https://emedicoz.com/testresult/683510s14692547 20/189
6/9/24, 12:45 PM Neet PG Preparation, Neet PG Coaching, FMGE, USMLE

Correct Answer (B)

Significantly poorer hearing loss in one ear

Ref: Read the text below

Sol:

• If only one ear has hearing loss, and it can be effectively fitted with a hearing
aid, then a monaural fitting is indicated.

• If the poorer ear cannot be effectively fitted, there are several methods of
delivering a signal from the impaired side to the better hearing ear.

• This approach is generally referred to as contralateral routing of signals


(CROS). In the typical model of CROS amplification, a microphone/transmitter is
worn on the unaidable ear, and a receiver is worn on the normal ear where
sound is delivered via a loudspeaker.

• When the hearing is also decreased in the better ear, the receiver device on
the better ear can be integrated with or coupled to a traditional hearing aid. This
allows the signal presented to the better ear to be amplified as needed.

• This is known as a bilateral CROS (BiCROS). The use of CROS or BiCROS


devices does not allow patients to localize sound

Question: 22
Which of the following statements is true about inverted papilloma
arising from the lateral wall of nose?

https://emedicoz.com/testresult/683510s14692547 21/189
6/9/24, 12:45 PM Neet PG Preparation, Neet PG Coaching, FMGE, USMLE

A Polypoidal masses
resemble allergic nasal polyps

B Affects females in
adolescent period

C Sends early distant


metastases

D Treatment requires
maxillectomy with postoperative
radiotherapy

37.22% People got this right

Explanation:

Correct Answer (A)

Polypoidal masses resemble allergic nasal polyps

Ref.: Read the text below

Sol :

• Inverted papilloma, which arises from the mucosa of lateral wall of nose,
most frequently affects males in the age group of 30-5O years.

• Only 10-15% are associated with malignancy.

• Treatment is complete excision which amounts to medial maxillectomy.

• Post-operative radiotherapy is not necessary.

Question: 23
Asymmetrical Congestive Response (The Nasal Cycle) occurs every:-

https://emedicoz.com/testresult/683510s14692547 22/189
6/9/24, 12:45 PM Neet PG Preparation, Neet PG Coaching, FMGE, USMLE

A 1-2 hours

B 2–4 hours

C 1-2 mins

D 2–7 mins

39.83% People got this right

Explanation:

Correct Answer (B)

2–4 hours

Sol:

Regulation Response Types

1. Asymmetrical Congestive Response (The Nasal Cycle): Normal physiological


congestion/decongestion cycle alternating between nasal sides every 2–4 hours

2. Symmetrical Congestive Response: temporary bilateral congestion induced by


exercise, changes in body position, hyperventilation, cold air, sulfur, histamine,
and other irritants; lasts 15–30 minutes

Question: 24
After a temporal bone trauma, where is the most common injury site to
the ossicular chain?

A Incudostapedial joint >


dislocation of the incus > fracture
of the stapes crura > fixation of
the ossicles in the epitympanum
> fracture of the malleus

https://emedicoz.com/testresult/683510s14692547 23/189
6/9/24, 12:45 PM Neet PG Preparation, Neet PG Coaching, FMGE, USMLE

B Dislocation of the incus >


incudostapedial joint > fracture of
the stapes crura > fixation of the
ossicles in the epitympanum >
fracture of the malleus

C Fracture of the stapes crura


> incudostapedial joint >
dislocation of the incus > fixation
of the ossicles in the
epitympanum > fracture of the
malleus

D Incudostapedial joint >


fracture of the malleus>
dislocation of the incus > fracture
of the stapes crura > fixation of
the ossicles in the epitympanum

24.23% People got this right

Explanation:

Correct Answer (A)

Incudostapedial joint > dislocation of the incus > fracture of the stapes crura >
fixation of the ossicles in the epitympanum > fracture of the malleus

Ref: Read the text below

Sol:

• The most common injuries to the ossicular chain are separation of the
incudostapedial joint (82%), dislocation of the incus (57%), and fracture of the
stapes crura (30%).

• Fixation of the ossicles in the epitympanum (25%) and fracture of the malleus
(11%) occur less frequently.

https://emedicoz.com/testresult/683510s14692547 24/189
6/9/24, 12:45 PM Neet PG Preparation, Neet PG Coaching, FMGE, USMLE

Question: 25
At what age the lower end of radius ossifies in a female?

A 16-17 years

B 17-18 years

C 18-19 years

D 19-20 years

23.32% People got this right

Explanation:

Correct Answer (B)

Question: 26
Medical Termination of Pregnancy Act has been last amended in which
year

A 2003

B 2014

C 2017

D 2021

56.98% People got this right

Explanation:

https://emedicoz.com/testresult/683510s14692547 25/189
6/9/24, 12:45 PM Neet PG Preparation, Neet PG Coaching, FMGE, USMLE

Correct Answer (D)

Question: 27
Bullet fingerprinting refers to

A Detection of fingerprints
on bullet

B Comparison of test bullet


and crime bullet

C Comparison of primary and


secondary markings on crime
bullet

D None of the above

20.2% People got this right

Explanation:

Correct Answer (B)

Question: 28
A patient from West Bengal presents to casualty with pigmentation
changes of skin, hard patches on palms and soles, basal cell carcinoma of
skin. What could be the possible cause?

A Arsenic poisoning

B Lead poisoning

https://emedicoz.com/testresult/683510s14692547 26/189
6/9/24, 12:45 PM Neet PG Preparation, Neet PG Coaching, FMGE, USMLE

C Cadmium poisoning

D Zinc poisoning

45.07% People got this right

Explanation:

Correct Answer (A)

Question: 29
Which is not a correct match

A Hydrogen sulphide – rotten


egg like smell

B Water hemlock – carrot like


smell

C Zinc phosphide – garlic like


smell

D Nitrobenzene – shoe polish


like smell

14.76% People got this right

Explanation:

Correct Answer (C)

Question: 30

https://emedicoz.com/testresult/683510s14692547 27/189
6/9/24, 12:45 PM Neet PG Preparation, Neet PG Coaching, FMGE, USMLE

A 2-days old neonate born to a 24-year-old mother without adequate


prenatal care is admitted in NICU with bulging fontanelle and intense
crying. On physical examination, The neonate is hypothermic, tachycardic,
and tachypneic, and has mottled skin. Blood samples are obtained, and
empiric antibiotics are initiated for the treatment of sepsis. Culture on blood
agar grows bacterial colonies with a narrow zone of clear hemolysis. A
special test shown in the image is revealed the pathogen. Which of the
following is the most likely pathogen infecting this neonate?

A Enterococcus faecalis

B Staphylococcus
epidermidis

C Streptococcus agalactiae

D Streptococcus pneumoniae

45.2% People got this right

Explanation:

Correct Answer (C)

Streptococcus agalactiae

• Streptococcus agalactiae (group B Streptococcus [GBS]) is often a normal


commensal of the genitourinary and gastrointestinal tracts of pregnant women.

https://emedicoz.com/testresult/683510s14692547 28/189
6/9/24, 12:45 PM Neet PG Preparation, Neet PG Coaching, FMGE, USMLE

• Those who are colonized and do not receive intrapartum antibiotic


prophylaxis can transmit the bacteria to the fetus during passage through the
vaginal canal, leading to neonatal sepsis (eg, irritability, lethargy, grunting,
tachypnea, temperature instability, hypotension) within hours of birth.

• GBS is a leading cause of early neonatal sepsis and can be identified by the
following laboratory features:

- Narrow zone of beta-hemolysis (clear) when plated on blood agar.

• Production of CAMP factor, a phospholipase that enhances the activity of


beta-hemolysins secreted by certain strains of Staphylococcus aureus.

• Testing for CAMP factor (CAMP test) can confirm the presence of GBS; this
test plates suspected GBS colonies perpendicular to an appropriate S. aureus
strain and evaluates for enhanced areas of hemolysis (arrow-shaped zones in the
image).

• GBS is also catalase-negative (as are all streptococci), contains the Lancefield
group B antigen in its cell wall, and hydrolyzes Hippurate.

Question: 31
A 32 year patient spontaneously delivers a stillborn fetus with multiple
congenital malformations. Fetal autopsy reveals microcephaly with thin
cerebral cortices, ventriculomegaly, and subcortical calcifications. Viral
RNA of a neurotropic virus is detected in body fluids of the mother and
the fetus. Transplacental transmission of this virus causes apoptosis of
neural progenitor cells in the developing fetus, leading to disruption of
neuronal proliferation, migration, and differentiation. Name the vector
responsible for transmitting this viral disease?

A Culex mosquito

B Ades mosquito

C Xenopsylla cheopsis

https://emedicoz.com/testresult/683510s14692547 29/189
6/9/24, 12:45 PM Neet PG Preparation, Neet PG Coaching, FMGE, USMLE

D Simulium fly

34.75% People got this right

Explanation:

Correct Answer (B)

Ades mosquito

• It’s a case of Zika viral disease.

• Zika virus is a single-stranded RNA virus belonging to the Flavi viridiae.


Transmission occurs primarily via an Aedes mosquito bite, but infected patients
can also spread the virus via genital secretions.

• The neurotropic virus can cross the placenta and infect and destroy fetal
neural progenitor cells, causing congenital Zika syndrome and possible fetal
demise.

• Fetal brain development is impaired due to disruption of normal


proliferation, migration, and differentiation of neurons. Classic findings in
affected new-borns include microcephaly with facial features out of proportion
to head size, arthrogryposis (contractures), seizures, hypertonia, and ocular
abnormalities.

• Loss of brain mass (eg, cortical thinning, ventriculomegaly) as well as


subcortical calcifications are typically present.

• Diagnosis is confirmed by detection of Zika RNA (real-time reverse


transcriptase PCR) in serum, urine, or cerebrospinal fluid.

• The mainstay of treatment for surviving infants is supportive care with


management of feeding difficulties, hydrocephalus, and seizures.

Question: 32
Few cases of severe pharyngitis were reported in a community of
immigrants. The patients had thick pharyngeal exudates, neck swelling, and
difficulty swallowing. One of them died from severe heart failure.

https://emedicoz.com/testresult/683510s14692547 30/189
6/9/24, 12:45 PM Neet PG Preparation, Neet PG Coaching, FMGE, USMLE

The toxin responsible has a mechanism of action most similar to another


toxin produced by which of the following bacteria?

A Pseudomonas aeruginosa

B Staphylococcus aureus

C EHEC

D Shigella dysenteriae

29.93% People got this right

Explanation:

Correct Answer (A)

Pseudomonas aeruginosa

• Severe pharyngitis with exudates and cervical lymphadenopathy in a group


of people with unknown vaccination status should raise suspicion for respiratory
diphtheria infection.

• Diphtheria toxin can cause severe myocarditis and heart failure, which
occurred in one of the patients in the vignette.

• Diphtheria toxin acts in a similar manner to exotoxin A, which is produced by


Pseudomonas aeruginosa.

• P aeruginosa produces several extracellular products, including exotoxin A,


collagenase, elastase, fibrinolysin, phospholipase C, and DNAse. These
substances assist in its invasion and dissemination in human tissues.

• Although they are structurally different, both diphtheria toxin and exotoxin A
ribosylate and inactivate elongation factor-2 (EF-2), halting human cell protein
synthesis and causing cell death.

https://emedicoz.com/testresult/683510s14692547 31/189
6/9/24, 12:45 PM Neet PG Preparation, Neet PG Coaching, FMGE, USMLE

• Exotoxin A is a major virulence factor and is responsible for the high


mortality associated with P aeruginosa septicaemia.

Question: 33
A 30-year-old woman comes to the emergency department with leg
swelling. Her dog, which is fully vaccinated and has lived with her for the
past year, bite her on the left leg last night, leading to a break in the skin
and bleeding. The patient irrigated the wound with bottled water and
soap and managed her pain with paracetamol. This morning, the wound
area became warm, red, and swollen. On examination, temperature is
36.40 C. There is an open wound on the left leg, with surrounding
erythema, warmth, and tenderness. No wound drainage or crepitus is
present. The distal pulses are palpable. Wound cultures grow gram-
negative coccobacilli; the culture has a mouse-like odor. Which of the
following organisms is most likely responsible for this patient's current
presentation?

A Bartonella henselae

B Campylobacter jejuni

C Clostridium perfringens

D Pasteurella multocida

32.38% People got this right

Explanation:

Correct Answer (D)

Pasteurella multocida

• This patient has a skin and soft-tissue infection (SSTI) that developed at the
site of a dog bite relatively soon following the bite.

https://emedicoz.com/testresult/683510s14692547 32/189
6/9/24, 12:45 PM Neet PG Preparation, Neet PG Coaching, FMGE, USMLE

• Pasteurella multocida is an organism found in the mouths of dogs and is


responsible for the majority of acute skin infections following a dog bite.

• Infection typically occurs within 24 hours of inoculation and has a


characteristic mouse-like odor (Indole-positive species).

• P multocida SSTI can also occur following a cat bite. Other organisms
associated with dog bites include staphylococci, Streptococci, and
Capnocytophaga canimorsus.

• Management includes wound care and antibiotics (eg, amoxicillin-


clavulanate)

Question: 34
A neonate born to a 26-year-old woman is evaluated in the new-born
nursery. The patient was delivered vaginally at 38 weeks gestation, and
Apgar scores were 8 and 9 at 1 and 5 minutes, respectively. Initial
evaluation shows hydrocephalus, jaundice, and hepatosplenomegaly.
Funduscopic examination reveals retinal exudates and scarring.
Histopathologic evaluation of the placenta shows infiltration of
lymphocytes, plasma cells, and macrophages, as well as many areas of
calcification. Within the placental tissue, there are numerous intracellular,
crescent-shaped organisms with a central nucleus as shown in the image.
Which of the following is the infective stage for the pathogen responsible
for this condition?

A Bradycyst

https://emedicoz.com/testresult/683510s14692547 33/189
6/9/24, 12:45 PM Neet PG Preparation, Neet PG Coaching, FMGE, USMLE

B Oocyst

C Tachyzoite

D Sporozoite

30.8% People got this right

Explanation:

Correct Answer (C)

Tachyzoite

• It’s a case of congenital toxoplasmosis

• Classical triad is Chorioretinitis, Diffuse intracranial calcifications and


Microcephaly (brain atrophy) or macrocephaly (hydrocephalus).

• This neonate with retinal lesions (chorioretinitis) and hydrocephalus has a


placenta containing inflammatory cells, calcifications, and crescent-shaped
organisms. These findings are consistent with congenital toxoplasmosis, an
infection caused by the intracellular parasite Toxoplasma gondii.

• All the stages are Infective stage to humans.

• Oocyst – Cat feces – Ingestion

• Bradycyst – Meat – Ingestion

• Tachyzoite – Circulation – Placental transfer, Blood transfusion.

Question: 35
A 21-year-old man is brought to the emergency department in a
stuporous condition. He returned from a visit to rural village 2 weeks ago.
One week ago, the patient began developing high-grade fever,
headaches, severe myalgias, and joint pains. He had similar symptoms
following a trip to the same village 4 years ago, but at that time the

https://emedicoz.com/testresult/683510s14692547 34/189
6/9/24, 12:45 PM Neet PG Preparation, Neet PG Coaching, FMGE, USMLE

symptoms did not progress and instead resolved spontaneously within a


few days. Temperature is 38.30 C, blood pressure is 80/50 mm Hg, and
pulse is 128/min. Examination shows a diffuse maculopapular rash,
multiple purpuric lesions, and hepatomegaly. Laboratory studies shows
marked thrombocytopenia, leukopenia, and elevated liver
aminotransferases. Which of the following is the most likely mechanism
for this patient's current severe illness?

A Antigenic drift in the virus

B Blunted T-lymphocyte
response

C Infection with a different


viral serotype

D Antigenic shift

29.52% People got this right

Explanation:

Correct Answer (C)

Infection with a different viral serotype.

o This patient's presentation suggests dengue hemorrhagic fever (DHF).

o Dengue viruses are transmitted by the Aedes mosquito and are single-
stranded RNA viruses (genus Flavi virus) with 4 different serotypes (DENV1-4).

o Primary (first) infection can be asymptomatic or cause a self-limited disease


in most adults. Symptomatic disease can cause high-grade fever, headache,
retro-orbital pain, bleeding (eg, epistaxis, petechiae purpura), diffuse macular
rash, muscle and joint pain ("break-bone fever"), leukopenia, thrombocytopenia,
and elevated liver enzymes.

o Primary infection leads to lifelong immunity against the same serotype, but
individuals can be infected with a different serotype. Secondary infection with a

https://emedicoz.com/testresult/683510s14692547 35/189
6/9/24, 12:45 PM Neet PG Preparation, Neet PG Coaching, FMGE, USMLE

different viral serotype can cause a more severe illness, possibly due to
antibody-dependent enhancement of infection, enhanced immune complex
formation, and/or accelerated T-lymphocyte responses.

Question: 36
A 3-year-old boy is being evaluated for persistent diarrhea. Although the
patient seemed healthy at his 12- month well child visit, since then he has
experienced 4 episodes of otitis media and 3 episodes of pneumococcal
pneumonia. He was at the 50th percentile for weight and height at 12
months but is now at the 25th percentile for height and 10th percentile
for weight. The patient is referred for upper gastrointestinal endoscopy,
and Giardia lamblia is demonstrated from duodenal aspirates. Further
workup shows very low serum levels of all immunoglobulin types. Flow
cytometry of this patient's peripheral blood is most likely to show a near
absence of cells bearing which of the following markers?

A CD4

B CD8

C CD16

D CD19

23.23% People got this right

Explanation:

Correct Answer (D)

CD19

• This patient's recurrent sinopulmonary infections, Giardia lamblia


gastroenteritis, failure to thrive, and low immunoglobulin levels are suggestive of
Bruton’s X-linked agammaglobulinemia (XLA).

https://emedicoz.com/testresult/683510s14692547 36/189
6/9/24, 12:45 PM Neet PG Preparation, Neet PG Coaching, FMGE, USMLE

• In this condition, a mutation in the Bruton tyrosine kinase gene causes failure
of bone marrow pre-B cells to develop into mature B cells, a step necessary for B
cells to leave the bone marrow and enter the peripheral circulation.

• Flow cytometry can be used to assess the number of circulating B cells by


using fluorescent tags that bind it to specific B cell surface proteins such as
CD19 and CD20

Question: 37
A 42-year-old man comes to the physician because of dysuria and blood in
his urine at the end of urination. He returned to the India 2 weeks ago after
several months touring Africa. While there. he went spelunking, bathed in
the beaches and lakes. and ate the local street food. Physical examination is
unremarkable. Ultrasonography reveals mild bilateral hydronephrosis and
bladder wall thickening. Urine microscopy revealed the pathogen as shown
in the image. Which of the following animals is the most likely source of this
patient's infection?

A Snail

B Fish

C Crab

https://emedicoz.com/testresult/683510s14692547 37/189
6/9/24, 12:45 PM Neet PG Preparation, Neet PG Coaching, FMGE, USMLE

D Aquatic plant

40.18% People got this right

Explanation:

Correct Answer (A)

Snail.

• This patient most likely has schistosomiasis caused by Schistosoma


haematobium.

• Humans acquire schistosomiasis via contact with infected freshwater.

• Freshwater snails release the cercariae into the environment.

• The larvae can penetrate intact human skin and causes Schistosomiasis.

• S. haematobium eggs tend to pierce the vesical and ureteral walls and be
expelled in the urine causes hematuria. Eggs with terminal spine is diagnostic od
S. hematobium.

Question: 38
A 18-year-old boy is returned from US hospitalized with recent-onset
insomnia, headaches. periodic agitation, and dysphagia. He is a high
school student interested in becoming a veterinarian. He owns a snake,
fish and parrot and about 4 weeks ago, went on a field trip to study bats
in a cave. The patient has no known medical problems and is not taking
any medications. He does not consume alcohol and has never used illicit
drugs. Despite hospitalization and treatment, he develops painful spasms,
progressive paralysis, coma and dies. Which of the following
interventions would have most likely prevented this student from
developing this illness?

A Cellular receptor inhibitor


drug

https://emedicoz.com/testresult/683510s14692547 38/189
6/9/24, 12:45 PM Neet PG Preparation, Neet PG Coaching, FMGE, USMLE

B Inactivated vaccine

C Live attenuated vaccine

D Toxoid vaccine

17.57% People got this right

Explanation:

Correct Answer (B)

Inactivated Vaccine

• Restlessness, agitation, and dysphagia progressing to coma within weeks of


exploring a bat cave is concerning for rabies encephalitis.

• Although bats (and as, commonly, raccoons) are among the main sources of
rabies in the United States.

• Many patients with bat rabies are not aware that they had been bitten.

• Once in a wound. the virus travels retrograde via peripheral nerves to the
dorsal root ganglia and then to the brain, where replication occurs.

• Common manifestations include a nonspecific, flu-like prodrome followed by


an acute neurologic syndrome with agitation, persistent fever, variable
consciousness and painful spasms with swallowing or inspiration (causing
dysphagia, hydrophobia, and aerophobia), generalized flaccid paralysis and
coma. Most patients dying within weeks.

• Prophylactic vaccination is recommended for individuals at high exposure


risk like veterinarians: cave explorers. laboratory workers handling infected
tissues, travellers to countries where rabid dogs are prevalent).

• Postexposure prophylaxis {a medical emergency] includes rabies immune


globulin and vaccination: it is not effective after symptom onset.
The approved rabies vaccines consist of various rhabdovirus strains grown in
tissue cell culture and inactivated by beta-propiolactone.

https://emedicoz.com/testresult/683510s14692547 39/189
6/9/24, 12:45 PM Neet PG Preparation, Neet PG Coaching, FMGE, USMLE

Question: 39
A 21-year-old previously healthy man comes to the clinic due to fever.
Malaise, myalgia, and non-productive cough. His symptoms began 2
weeks after returning from exploring caves in Tamil Nadu. The patient
does not use tobacco, alcohol, or illicit drugs. Temperature is 38.30 C.
blood pressure is 120/70 mm Hg, pulse is 96/min, and respirations are
15/min. Physical examination reveals lung crackles. Chest x-ray shows
patchy parenchymal infiltrates with enlarged mediastinal and hilar lymph
nodes. Which of the following is most likely to be present in this patient‘s
pulmonary lesions?

A Budding yeasts with a thick


capsule

B Macrophages with
intracellular small yeasts

C Round spherules with


multiple endospores

D Septate hyphae with


dichotomous branching

17.99% People got this right

Explanation:

Correct Answer (B)

Macrophages with intracellular small yeasts

• Histoplasma capsulatum is a dimorphic fungus.

• It exists as a filamentous mold in the environment and is found primarily in


soil contaminated with bird or bat droppings. Therefore, infected patients often
report a history of exploring caves (exposure to bats) or cleaning bird cages or
coops.

https://emedicoz.com/testresult/683510s14692547 40/189
6/9/24, 12:45 PM Neet PG Preparation, Neet PG Coaching, FMGE, USMLE

• H. capsulatum is transmitted when fungal spores from contaminated soil are


inhaled into the alveoli. At body temperature the spores germinate into yeasts
and are subsequently phagocytosed by alveolar macrophages. The pathogen
eludes macrophage-mediated destruction by preventing acidification of the
phagolysosome and initially replicates in an unchecked fashion.

• Therefore. light microscopy of the affected areas reveals macrophages with


intracellular ovoid or round yeast cells.

Question: 40
Most common age related change in vitreous-

A Anterior vitreous
detachment

B Posterior vitreous
detachment

C Vitreous haemorrhage

D Vitritis

44.37% People got this right

Explanation:

Correct Answer (B)

Over time, the vitreous gel that fills the eye becomes liquid and condenses
(shrinks) due to age and normal wear and tear. Eventually it cannot fill the whole
volume of the eye’s vitreous cavity (which remains the same size during
adulthood) and so the gel separates from the retina, located at the very back of
the eye cavity.

Over the next 1 to 3 months, the vitreous gel further condenses and the sides of
the gel also separate from the retina until the PVD is complete and the vitreous

https://emedicoz.com/testresult/683510s14692547 41/189
6/9/24, 12:45 PM Neet PG Preparation, Neet PG Coaching, FMGE, USMLE

gel is attached to the retina only at the vitreous base. Posterior vitreous
detachment is rare in people under the age of 40, and increasingly common
during advanced age. Additional risk factors for PVD include myopia
(nearsighted- ness), trauma, and recent eye surgery such as a cataract operation

Question: 41
Diplopia in right lateral rectus palsy is maximum in?

A Vertical diplopia on
downward gaze

B Vertical diplopia on upward


gaze

C Horizontal diplopia on
inward gaze

D Horizontal diplopia on
outward gaze

45.44% People got this right

Explanation:

Correct Answer (D)

Horizontal diplopia- maximum in direction of paralysed muscle

Question: 42
What is the refractive error here?

https://emedicoz.com/testresult/683510s14692547 42/189
6/9/24, 12:45 PM Neet PG Preparation, Neet PG Coaching, FMGE, USMLE

A Simple myopic astigmatism

B Simple myopia

C Compound myopic
astigmatism

D Compound hypermetropic
astigmatism

47.14% People got this right

Explanation:

Correct Answer (C)

Both rays are in front of retina at different points

Question: 43
All other findings can be seen in the patient with the following lesion except

https://emedicoz.com/testresult/683510s14692547 43/189
6/9/24, 12:45 PM Neet PG Preparation, Neet PG Coaching, FMGE, USMLE

A Optic nerve glioma

B Eyelid tumour

C Choroidal hemangioma

D Sphenoid dysplasia

17.57% People got this right

Explanation:

Correct Answer (C)

Lisch nodules in neurofibromatosis

Choroidal hemangioma is seen in Stuge weber syndrome (tomato ketchup


appearance)

Question: 44
True regarding tarsal plate are all except?

A Acts a skeleton for eyelids

https://emedicoz.com/testresult/683510s14692547 44/189
6/9/24, 12:45 PM Neet PG Preparation, Neet PG Coaching, FMGE, USMLE

B Wider in upper eyelids

C Have meibomian glands

D Attached to lateral
palpebral superioris ligament in
lower eyelid

37.76% People got this right

Explanation:

Correct Answer (D)

Tarsal plates are responsible for forming skeleton of the eyelids and are rixh in
meibomian glands. The superior tarsal plate is larger, wider and semilunar, while
the inferior tarsal plate is smaller, thinner and elliptical in shape. LPS tendon is
attached to the upper tarsal plate.

Question: 45
A patient comes to OPD with severe pain in the left eye. He uses contact
lens frequently. Following was the examination on slit lamp. What would be
the culture used to identify the organism?

A Theyer-Martin

https://emedicoz.com/testresult/683510s14692547 45/189
6/9/24, 12:45 PM Neet PG Preparation, Neet PG Coaching, FMGE, USMLE

B Sabouraud's agar

C Lowenstein-Jensen

D Non-nutrient E.coli-
enriched agar

32.05% People got this right

Explanation:

Correct Answer (D)

Non-nutrient E.coli-enriched agar is used to cultivate acanthoemeba. There is


discrepancy in signs and symptom – more symptoms than signs.

Question: 46
What could be the profile of the patient?

A Old female with watering

B Young female with itching

C Young boy with itching

https://emedicoz.com/testresult/683510s14692547 46/189
6/9/24, 12:45 PM Neet PG Preparation, Neet PG Coaching, FMGE, USMLE

D Old male with itching

38.97% People got this right

Explanation:

Correct Answer (C)

Vernal keratoconjunctivitis (VKC) is a recurrent bilateral disorder in which both


IgE- and cell-mediated immune mechanisms play important roles. It primarily
affects boys and onset is generally from about the age of 5 years onwards.
consist of intense itching, which may be associated with lacrimation,
photophobia, a foreign body sensation, burning and thick mucoid discharge.

Question: 47
The pictures are before and after Valsalva. What is likely condition?

A Orbital hemangioma

B Orbital varices

C Carotico-cavernous fistula

D Orbital dermoid

https://emedicoz.com/testresult/683510s14692547 47/189
6/9/24, 12:45 PM Neet PG Preparation, Neet PG Coaching, FMGE, USMLE

23.77% People got this right

Explanation:

Correct Answer (B)

Primary orbital varices (combined venous–lymphatic malformations of the orbit -


consist of a plexus of thinwalled distensible low-flow vein-like vessels that are
commonly. They are probably hamartomatous (hamartoma – a disorganized
overgrowth of mature tissues normally present in the involved area).

Question: 48
Which cataract surgery step is being done?

A Capsulorhhexis

B Hydrodissection

C IOL implantation

D Lens aspiration

40.36% People got this right

Explanation:

Correct Answer (A)

https://emedicoz.com/testresult/683510s14692547 48/189
6/9/24, 12:45 PM Neet PG Preparation, Neet PG Coaching, FMGE, USMLE

Making an opening in anterior lens capsule is called capsulorhhexis

Question: 49
A hypermetropic patient with closed angles on gonioscopy has IOP of 15
mm Hg with normal optic disc with normal visual fields. Diagnosis?

A Primary angle closure


suspect

B Primary angle closure

C Primary angle closure


glaucoma

D Ocular hypertension

33.36% People got this right

Explanation:

Correct Answer (A)

When angles are occludable/closed, IOP is less than 21 mm Hg and normal optic
disc and visual fields, its PACS

Question: 50
Ciprofloxacin should not be given to an asthmatic using theophylline
because:

A Theophylline inhibits
ciprofloxacin metabolism

https://emedicoz.com/testresult/683510s14692547 49/189
6/9/24, 12:45 PM Neet PG Preparation, Neet PG Coaching, FMGE, USMLE

B Ciprofloxacin decreases
effect of theophylline

C Ciprofloxacin inhibits
theophylline metabolism

D Theophylline induces
metabolism of ciprofloxacin

30.66% People got this right

Explanation:

Correct Answer (C)

Theophylline is metabolized CYP1A2, CIPROFLOXACIN is the microsomal enzyme


inhibitor, thereby ciprofloxacin inhibits the metabolism of theophylline and
increase plasma level of theophylline.

Question: 51
Voriconazole is not effective against

A Candida albicans

B Mucormycosis

C Candida tropicalis

D Aspergillosis

36.8% People got this right

Explanation:

Correct Answer (B)

https://emedicoz.com/testresult/683510s14692547 50/189
6/9/24, 12:45 PM Neet PG Preparation, Neet PG Coaching, FMGE, USMLE

Voriconazole is the DOC- invasive aspergillosis. And also useful in Fluconazole


resistant Candida & Febrile neutropenia

Fluconazole, voriconazole, and itraconazole do not have reliable activity against


mucormycosis

For mucormycosis – DOC is Intravenous liposomal amphotericin B and other


antifungal effective are Posaconazole & Isavuconazole

Question: 52
Match the following

A A-4, B-5, C.-2, D-1

B A-3, B-5, C.-2, D-1

C A-3, B-1, C.-4, D-2

D A-2, B-5, C.-6, D-2

35.71% People got this right

Explanation:

Correct Answer (C)

Tesamorelin GHRF analogue useful in HIV with lipodystrophy

https://emedicoz.com/testresult/683510s14692547 51/189
6/9/24, 12:45 PM Neet PG Preparation, Neet PG Coaching, FMGE, USMLE

Pasereotide stomatostain analogye useful in cushing syndrome

Teduglutide GLP 2 analogue useful in short bowel syndrome

Abaloparatide parathormone analogue useful in osteoporosis

Question: 53
Thalidomide is not used in

A HIV related neuropathy

B Erythema nodosum
leprosum

C HIV related oral ulcer

D Behcet’s disease

21.65% People got this right

Explanation:

Correct Answer (A)

Peripheral neuropathy is an adverse effect of thalidomide, therefore it is not


indicated for HIV induced peripheral neuropathy.

Clinical uses of thalidomide

AIDS related aphthous ulcers • AIDS related wasting syndrome • Multiple


myeloma and other solid tumors like AIDS-related Kaposi’s sarcoma • Prevention
of graft versus host disease (GVHD) after transplantation • Rheumatoid arthritis •
Ankylosing spondylitis • Crohn’s disease and Behcet’s syndrome

Adverse reactions to thalidomide

https://emedicoz.com/testresult/683510s14692547 52/189
6/9/24, 12:45 PM Neet PG Preparation, Neet PG Coaching, FMGE, USMLE

Teratogenicity • Peripheral neuropathy • Drowsiness • Skin rashes • Constipation

Question: 54
Which of the following pairs are incorrectly matched:

(a) Ocrilizumab – Multiple sclerosis

(b) Burosumab – x linked hypophosphatemia

(c) Yedolizumab- urothelial cancer

(d) Trastuzumab – her/2/neu +ve breast carcinoma

(e) Palivizumab – Psoriasis

A B&C

B A&C

C B&E

D C&E

15.48% People got this right

Explanation:

Correct Answer (D)

Ocrelizumab – targeted against CD20 useful for multiple sclerosis,

Burosumab targeting against FGF 23 useful in x linked hypophosphatemia,


vedolizumab- alpha 4beta7 integrin inhibitor useful in inflammatory bowel
disease, Trastuzumab is a monoclonal antibody against her 2/neu gene. It is used
for breast carcinoma.

Palivizumab is a monoclonal antibody used against respiratory syncytial virus.

https://emedicoz.com/testresult/683510s14692547 53/189
6/9/24, 12:45 PM Neet PG Preparation, Neet PG Coaching, FMGE, USMLE

Question: 55
Correctly matched pair of heavy metals and their chelating agents is:

A Iron-BAL

B Mercury-Calcium disodium
edetate

C Copper- trientine

D Aresenic-Desferrioxamine

37.85% People got this right

Explanation:

Correct Answer (C)

Lead, arsenic and mercury – antidote BAL

For iron poisoning antidotes are= desferrioxamine and deferiprone

For copper poisoning- zinc sulphate, d Penicillamine and trietntine

Question: 56
Drug recently approved for Amyotrophic Lateral Sclerosis

(a) Edaravone

(b) Piracetam

(c) Riluzole

(d) Mitomycin

https://emedicoz.com/testresult/683510s14692547 54/189
6/9/24, 12:45 PM Neet PG Preparation, Neet PG Coaching, FMGE, USMLE

A A only

B A&D

C A&C

D A, C & D

41.85% People got this right

Explanation:

Correct Answer (C)

Drugs approved for amyotrophic lateral sclerosis are edaravone (anti oxidant)
and riluzole( NMDA blocker)

Question: 57
A patient present with diabetic macular edema with glaucoma. Which of
the following drug should be used last for this patient?

A Alpha agonist

B Acetazolamide

C Beta blockers

D Prostaglandin analogue

22.86% People got this right

Explanation:

https://emedicoz.com/testresult/683510s14692547 55/189
6/9/24, 12:45 PM Neet PG Preparation, Neet PG Coaching, FMGE, USMLE

Correct Answer (D)

Prostaglandin analogue causes adverse effects of macular edema

Question: 58
High plasma protein binding of a drug result in:

A Increase in Vd

B Decrease in glomerular
filtration

C Increase in tubular
secretion

D Decreased drug
interactions

41.41% People got this right

Explanation:

Correct Answer (B)

Clinical Importance of Plasma Protein Binding

Duration of action: Drugs with high PPB are usually long acting

Distribution: High PPB drugs stay in plasma, thus have low Vd.

Displacement: Highly PPB drug can be displaced by another highly bound drug

Dialysis: It is not effective for drugs having high PPB ,as proteins cannot be
filtered by glomerulus, so if a drug has high binding to plasma proteins, it tends
to have less glomerular filtration.

https://emedicoz.com/testresult/683510s14692547 56/189
6/9/24, 12:45 PM Neet PG Preparation, Neet PG Coaching, FMGE, USMLE

Question: 59
Find out the true and false statements

A. Sacubitril inhibits both vasopeptidase and angiotensin converting


enzyme

B. Ranolazine aggravates movement disorder

C. Labetalol has norepinephrine reuptake inhibitor action

D. Treimethoprim causes hyperkalemia

E. Dofetilitde has least risk of causing QT prolongation

A A-T, B-T, C.-T, D-F, E-T

B A-T, B-T, C.-T, D-F, E-F

C A-F, B-T, C.-T, D-T, E-F

D A-T, B-T, C.-F, D-F, E-T

15.46% People got this right

Explanation:

Correct Answer (C)

Sacubitril inhibits vasopeptidase (neprilysin)

Ranolazine an antioxidant useful in angina may aggravates the risk of movement


disorder

Labetalol a beta [lus alpha blocker has norepinephrine reuptake inhibitory action

Trimethorprim has ENac blocking property, so risk of causing hyperkalemia

https://emedicoz.com/testresult/683510s14692547 57/189
6/9/24, 12:45 PM Neet PG Preparation, Neet PG Coaching, FMGE, USMLE

Dofetilite is the class III antiarrhythmic drug has more risk of causing QT
prolongation.

Question: 60
All are true about tourette’s syndrome except?

A Motor tics and vocal tics


both are seen in all cases

B More common in females

C Associated with OCD

D Neuroleptics are useful in


the treatment

15.18% People got this right

Explanation:

Correct Answer (B)

Tourette syndrome is more common in males than females. It is commonly


associated with OCD and neuroleptics (antipsychotics) are used in treatment.

Clinical features (Tetrad of Tourette syndrome):

1. Motor tics

2. Vocal tics

3. Coprolalia (obscene words)

4. Palilalia (repeating words)

https://emedicoz.com/testresult/683510s14692547 58/189
6/9/24, 12:45 PM Neet PG Preparation, Neet PG Coaching, FMGE, USMLE

Question: 61
A patient who has taken lithium for some months discontinues it and says
‘the ocean needs a sail as rat needs a tail, so write your exam and don’t
fail, results will be out in a mail’. Which of the following symptoms is he
exhibiting?

A Flight of ideas

B Tangentiality

C Neologisms

D Clang associations

52.16% People got this right

Explanation:

Correct Answer (D)

Clang associations

Sail-tail-fail-mail (rhyming is observed which is characteristic of clang


associations)

Not flight of ideas, because no specific one word connection

Not tangentiality, because no evidence of losing the track

Not Neologisms, because no new words used

Question: 62
Maximum duration for which a person can take himself voluntary
admission under the mental health act? (*)

https://emedicoz.com/testresult/683510s14692547 59/189
6/9/24, 12:45 PM Neet PG Preparation, Neet PG Coaching, FMGE, USMLE

A 14 days

B 48 hours

C 30 days

D 90 days

35.61% People got this right

Explanation:

Correct Answer (C)

30 days

Admission: The Mental health act 2017 allows two types of admissions

a. Independent admissions: When the patient himself wants to get admitted.


According to Section 86 of MHCA 2017, any person who considers
himself/herself to be mentally ill can request for admission in MHE under the
section. The MHP may admit such person if he/she feels the illness is severe and
the patient will benefit from admission. According to the act, the presence of
NR/caregiver is not mandatory during the IP course of an independent patient. If
admission lasts more than 30 days, it should be again informed to the board.

b. Supported admissions: A person who needs admission, however, has lost


the capacity to make mental healthcare or treatment decisions, and hence needs
high level of support from the nominated representative, can be admitted as a
‘supported admission’. The nominated representative gives consent for
admission in this case. The MHRB must be informed within 7 days of a
supported admission (3 days in the case of a minor or woman). The admitted
person, his/her NR, or an appropriate organization may appeal this decision. If a
Section 89 admission has to continue beyond its allowed maximum duration of
30 days and ongoing supported admission is required, this can be done under
Section 90. At this stage, the MHRB should be informed, and they must review
the admission within 21 days and either permit the admission or order discharge

https://emedicoz.com/testresult/683510s14692547 60/189
6/9/24, 12:45 PM Neet PG Preparation, Neet PG Coaching, FMGE, USMLE

of the individual. These reviews of a supported admission continue at a


maximum frequency of 180 days.

Question: 63
Most widely abused substance causing dependence? (*)

A Cannabis

B Cocaine

C Heroin

D Amphetamine

39.8% People got this right

Explanation:

Correct Answer (A)

Cannabis

Prevalence wise cannabis is more widely used then any other substance
mentioned here in options. (Ref : Substance use manual by prof lal)
“Cannabis is by far the most widely cultivated, trafficked and abused illicit drug.
Half of all drug seizures worldwide are cannabis seizures. The geographical
spread of those seizures is also global, covering practically every country of the
world. About 147 million people, 2.5% of the world population, consume
cannabis (annual prevalence) compared with 0.2% consuming cocaine and 0.2%
consuming opiates. In the present decade, cannabis abuse has grown more
rapidly than cocaine and opiate abuse.”

Question: 64

https://emedicoz.com/testresult/683510s14692547 61/189
6/9/24, 12:45 PM Neet PG Preparation, Neet PG Coaching, FMGE, USMLE

The term used in psychiatry to describe unreal sensation of mind, in


which the person has “out of self” experiences, with intact reality contact?

A Trance

B Fugue

C Depersonalization

D Dejavu

36.59% People got this right

Explanation:

Correct Answer (C)

Depersonalization

DSM-5 Diagnostic Criteria for Depersonalization Disorder

A. Persistent or recurrent experiences of feeling detached from, and as if one is


an outside observer of, one's mental processes or body (e.g., feeling like one is
in a dream).

B. During the depersonalization experience, reality testing remains intact.

C. The depersonalization causes clinically significant distress or impairment in


social, occupational, or other important areas of functioning.

D. The depersonalization experience does not occur exclusively during the


course of another mental disorder, such as schizophrenia, panic disorder, acute
stress disorder, or another dissociative disorder, and is not due to the direct
physiological effects of a substance (e.g., a drug of abuse, a medication) or a
general medical condition (e.g., temporal lobe epilepsy).

Question: 65
https://emedicoz.com/testresult/683510s14692547 62/189
6/9/24, 12:45 PM Neet PG Preparation, Neet PG Coaching, FMGE, USMLE

What is the most likely diagnosis from the options listed?

A Miliary tuberculosis

B Sarcoidosis

C Silicosis

D Varicella zoster

3.7% People got this right

Explanation:

Correct Answer (D)

Many small dense (calcified) pulmonary nodules without any lymphadenopathy


evident. The density and size of the nodules suggests healed varicella zoster
infection over the other options.

Question: 66
What is the most likely diagnosis?

https://emedicoz.com/testresult/683510s14692547 63/189
6/9/24, 12:45 PM Neet PG Preparation, Neet PG Coaching, FMGE, USMLE

A Medullary sponge kidney

B Putty kidney

C Von Hippel-Lindau disease

D Xanthogranulomatous
pyelonephritis

17.36% People got this right

Explanation:

Correct Answer (A)

Bilateral clustered renal calcifications corresponding to the medullary pyramids


with paintbrush appearance in keeping with medullary sponge kidney. Putty
kidney due to tuberculosis would be unilateral and result in a diffusely calcified
and small kidney. Von Hippel-Lindau disease is associated with renal cysts and
renal angiomyolipomas (which occasionally can calcify), and an increased risk
renal cell carcinoma. Xanthogranulomatous pyelonephritis is associated with
staghorn calculi.

Question: 67

https://emedicoz.com/testresult/683510s14692547 64/189
6/9/24, 12:45 PM Neet PG Preparation, Neet PG Coaching, FMGE, USMLE

Patient with cough and fever. CT chest is shown to you. What is the most
likely diagnosis

A Pleural pathology

B Parenchymal Pathology

C Mediastinal Pathology

D Pericardial pathology

55.35% People got this right

Explanation:

Correct Answer (B)

Typical tree in bud appearance is seen, of endobronchial spread of lung


parenchymal disease.

Question: 68
Identify the following image

https://emedicoz.com/testresult/683510s14692547 65/189
6/9/24, 12:45 PM Neet PG Preparation, Neet PG Coaching, FMGE, USMLE

A Chronic EDH with


rebleeding

B Chronic SDH with


rebleeding

C SAH

D Brain abscess

40.88% People got this right

Explanation:

Correct Answer (B)

Note the concavo-convex-bilateral bleeding with both hypo and hyperdense


components.

Question: 69
The patient in emergency room presents difficulty swallowing and drooling,
fever difficulty speaking , inspiratory stridor and severe dysphagia. His is
suspected to have upper airway compromise. His lateral neck xray is shown
below. What is the diagnosis

https://emedicoz.com/testresult/683510s14692547 66/189
6/9/24, 12:45 PM Neet PG Preparation, Neet PG Coaching, FMGE, USMLE

A Acute epiglottitis

B Tonsillar abcess

C Foreign body in pharynx

D Laryngeal polyp

59.75% People got this right

Explanation:

Correct Answer (A)

Lateral soft tissue radiograph of the neck demonstrates thickening of the


epiglottis and aryepiglottic folds referred to as the thumb sign.

Question: 70
Which of the following is not included in CHA2DS2Vasc score to assess
the risk of thromboembolism in atrial fibrillation?

A Congestive Heart Failure

B Hyperlipidemia

https://emedicoz.com/testresult/683510s14692547 67/189
6/9/24, 12:45 PM Neet PG Preparation, Neet PG Coaching, FMGE, USMLE

C Age > 75years

D Diabetes

42.85% People got this right

Explanation:

Correct Answer (B)

Hyperlipidemia

Ref: Read the text below

Sol:

CHA2DS2Vasc score was developed to assess the risk of thromboembolism in


non rheumatic atrial fibrillation. Scores were alloted to various risk factors as
follows:

Congestive heart failure: 1

History of hypertension: 1

Age > 75 : 2, Age -65-75years :1 point,

Diabetes : 1

Stroke / TIA : 2, Sex is Female:1 point

Vascular event :1

Warfarin is indicated when CHA2DS2Vasc score is >=2

Question: 71
Which one of the following is not true regarding homocystinuria?

A Downward dislocation of
lens

https://emedicoz.com/testresult/683510s14692547 68/189
6/9/24, 12:45 PM Neet PG Preparation, Neet PG Coaching, FMGE, USMLE

B Marked osteoporosis

C Stroke in young

D Autosomal dominant
inheritance

35.82% People got this right

Explanation:

Correct Answer (D)

Autosomal dominant inheritance

Ref.: Read the text below

Sol:

Homocystinuria:

Autosomal recessive inheritance.


Reduced activity of cystathion –synthatase, the pyridoxal phosphate-
dependent enzyme.
Most patients present between 3 and 5 years of age.
Mental retardation, osteoporosis, a characteristic dislocation of the lens and
have marfanoid features.
Life – threatening vascular complications (affecting coronary, renal and cerebral
arteries) can occur during the first decade of life.

Question: 72
In high dose dexamethasone test cortisol production will be unchanged
in:-

A ACTH secreting pituitary


tumor

https://emedicoz.com/testresult/683510s14692547 69/189
6/9/24, 12:45 PM Neet PG Preparation, Neet PG Coaching, FMGE, USMLE

B Ectopic ACTH production

C Both of these.

D Exogenous steroid use.

37.99% People got this right

Explanation:

Correct Answer (B)

Ectopic ACTH production

Ref: Read the text below

Sol:

• Suppression of cortisol production after high dose dexamethasone i.e 2 mg


i/v for 2 days occurs in Pituitary adenoma and hypothalamic disorders while no
suppression occurs in ACTH secreting ectopic tumors as well as adrenal tumors
causing Cushing syndrome.

Question: 73
What is the area marked ‘X’

A ST segment

https://emedicoz.com/testresult/683510s14692547 70/189
6/9/24, 12:45 PM Neet PG Preparation, Neet PG Coaching, FMGE, USMLE

B ST interval

C SU segment

D QT interval

36.99% People got this right

Explanation:

Correct Answer (B)

ST interval

Ref: Read the text below

Sol:

Question: 74
The following are true regarding Interferon Gamma release assays in
tuberculosis except

A Antigens from all BCG


strains and nontuberculous

https://emedicoz.com/testresult/683510s14692547 71/189
6/9/24, 12:45 PM Neet PG Preparation, Neet PG Coaching, FMGE, USMLE

mycobacteria are present in the


assay

B Specificity of the test is


superior to Tuberculin skin test in
BCG vaccinated patients

C Sensitivity is comparable to
Tuberculin skin test

D Sensitivity is reduced in
HIV infection with low CD4 count

14.5% People got this right

Explanation:

Correct Answer (A)

Antigens from all BCG strains and nontuberculous mycobacteria are present in
the assay

Ref: Read the text below

Sol:

DIAGNOSIS OF LATENT M. TUBERCULOSIS INFECTION

● Tuberculin Skin testing PPD (RT23)is used. The greatest limitation of PPD is its
lack of mycobacterial species specificity, a property due to the large number of
proteins in this product that are highly conserved in the various species.

- It probably measures the response to antigenic stimulation by T cells that


reside in the skin rather than the response of re circulating memory T cells.

-The test is of limited value in the diagnosis of active TB because of its relatively
low sensitivity and specificity and its inability to discriminate between LTBI and
active disease.

- False-positive reactions may be caused by infections with non tuberculous


mycobacteria and by BCG vaccination.

https://emedicoz.com/testresult/683510s14692547 72/189
6/9/24, 12:45 PM Neet PG Preparation, Neet PG Coaching, FMGE, USMLE

-Repeated TST can produce larger reaction sizes due to either boosting or true
conversion. The “boosting phenomenon” is a spurious TST conversion resulting
from boosting of reactivity on subsequent TST 1–5 weeks after the initial test.

● IFN-γ Release Assays:

- The T-SPOT TB and the QuantiFERON TB Gold are two in vitro assays that
measure T cell release of IFN-γ in response to stimulation with the highly TB-
specific antigens ESAT-6 and CFP-10

- These tests likely measure the response of recirculating memory T cells to


persisting bacilli producing antigenic signals.

● Potential advantages of IGRAs include

- More specific than the TST & less cross-reactivity due to BCG vaccination and
sensitization by NTM.

- The need for fewer patient visits to complete testing,

- Avoidance of subjective measurements such as skin induration.

- In view of higher costs and increased technical requirements, the WHO does
not recommend the replacement of the TST by IGRAs in low- and middle-
income countries.

- Because of the paucity of data on the use of IGRAs in children, the TST is
preferred for LTBI testing of children under age 5.

- IGRAs have only modest predictive value for incident active TB, are not useful
in identifying patients with the highest risk of progression toward disease, and
cannot be used for diagnosis of active TB.

Question: 75
Which of the following anti-arrhythmic drugs is least likely to cause
torsades de pointes?

A Amiodarone

https://emedicoz.com/testresult/683510s14692547 73/189
6/9/24, 12:45 PM Neet PG Preparation, Neet PG Coaching, FMGE, USMLE

B Quinidine

C Sotalol

D Procainamide

19.53% People got this right

Explanation:

Correct Answer (A)

Amiodarone

Ref: Read the text below

Sol:

• Though amiodarone can cause QT interval prolongation, it is least likely to


cause torsades de pointes.

• The proposed reason is that even though it is a class III anti-arrhythmic drug,
it has additional properties belonging to all other classes of action.

• Pure class III drugs are more pro-arrhythmic than amiodarone

Question: 76
A lesion at angular gyrus with an intact brocas and wernickes area leads
to

A Difficulty in understanding
written text

B Difficulty in speech output

C Difficulty in comprehension

https://emedicoz.com/testresult/683510s14692547 74/189
6/9/24, 12:45 PM Neet PG Preparation, Neet PG Coaching, FMGE, USMLE

D Difficulty in repetition.

20.51% People got this right

Explanation:

Correct Answer (A)

Difficulty in understanding written text

Ref– Read the text below

Sol:

Gerstmann's Syndrome

• The combination of acalculia (impairment of simple arithmetic), dysgraphia


(impaired writing), finger anomia (an inability to name individual fingers such as
the index and thumb), and right-left confusion (an inability to tell whether a
hand, foot, or arm of the patient or examiner is on the right or left side of the
body) is known as Gerstmann's syndrome.

• When Gerstmann's syndrome is seen in isolation, it is commonly associated


with damage to the inferior parietal lobule (especially the angular gyrus) in the
left hemisphere.

Question: 77
All of the following are used in the treatment or prevention of
gastrointestinal bleeding secondary to portal hypertension except:-

A Transjugular intrahepatic
portosystemic shunt

B Peritoneovenous jugular
shunt

C Sclerotherapy

https://emedicoz.com/testresult/683510s14692547 75/189
6/9/24, 12:45 PM Neet PG Preparation, Neet PG Coaching, FMGE, USMLE

D Vasoressin

42.39% People got this right

Explanation:

Correct Answer (B)

Peritoneovenous jugular shunt

Ref: Read the text below

Sol :

• Peritoneovenous jugular shunts are used to treat refractory ascites. They have
no role in treating variceal bleeding.

Question: 78
All the following are associated with pure red cell aplasia except

A Anterior mediastinal
masses

B Connective tissue disorders

C Giant pronormoblasts

D Low erythropoietin levels

23.53% People got this right

Explanation:

Correct Answer (D)

Low erythropoietin levels

Ref– Read the text below


https://emedicoz.com/testresult/683510s14692547 76/189
6/9/24, 12:45 PM Neet PG Preparation, Neet PG Coaching, FMGE, USMLE

Sol:

• Pure red cell aplasia is a normochromic, normocytic anemia with absent


erythroblasts on the bone marrow, hence the diminished number or lack of
reticulocytes.

• The bone marrow shows red cell aplasia and the presence of giant
pronormoblasts.

• Several conditions have been associated with pure red cell aplasia, including
viral infections such as B19 parvovirus (which can have cytopathic bone marrow
changes), HIV, EBV, HTLV, and hepatitis B virus; malignancies such as thymomas
and lymphoma (which often present with an anterior mediastinal mass);
connective tissue disorders such as SLE and rheumatoid arthritis (RA); pregnancy;
drugs; and hereditary disorders. Erythropoietin levels are elevated because of the
anemia.

Question: 79
Of the following the most sensitive test to differentiate prerenal from
intrinsic (renal) types of acute renal failure is

A The urinary sodium index

B The urine specific gravity

C The urine osmolality

D The fractional excretion of


sodium (FENa)

54.17% People got this right

Explanation:

Correct Answer (D)

https://emedicoz.com/testresult/683510s14692547 77/189
6/9/24, 12:45 PM Neet PG Preparation, Neet PG Coaching, FMGE, USMLE

The fractional excretion of sodium (FENa)

Ref.: Read the text below

Sol :

Although the other renal failure indices may offer some guidance as to the type
of acute renal failure, the FENa is considered to be the most sensitive of the
choices provided.

Question: 80
Evidence of infection with hepatitis B virus is seen in association with
which of the following disease states?

A Henoch–schönlein purpura

B Mixed cryoglobulinemia

C Wegener’s granulomatosis

D Sjögren’s syndrome

26.33% People got this right

Explanation:

Correct Answer (B)

Mixed cryoglobulinemia

Ref: Read the text below

Sol :

• Two thirds of patients with mixed cryoglobulinemia have evidence of


infection with hepatitis B virus.

https://emedicoz.com/testresult/683510s14692547 78/189
6/9/24, 12:45 PM Neet PG Preparation, Neet PG Coaching, FMGE, USMLE

• This virus also has been found in the serum of 30 to 40% of patients with
polyarteritis nodosa.

• Because hepatitis B virus has been found in vascular lesions along with
immunoglobulin and complement,vasculitis in these individuals might be due to
immune complexes containing the hepatitis antigen.

Question: 81
In achalasia, basic abnormality is :

A Inhibitory neural
degeneration localized to nerve
processes

B Inhibitory neural
degeneration localized to nerve
cell bodies

C Excitatory neural
degeneration localized to nerve
processes

D Excitatory neural
degeneration localized to nerve
cell bodies

29.45% People got this right

Explanation:

Correct Answer (B)

Inhibitory neural degeneration localized to nerve cell bodies

Ref: Read the text below

Sol :

https://emedicoz.com/testresult/683510s14692547 79/189
6/9/24, 12:45 PM Neet PG Preparation, Neet PG Coaching, FMGE, USMLE

In achalasia, the basic abnormality is due to inhibitory neural degeneration


localized to nerve cell bodies.

Question: 82
Which of the following agents causes non inflammatory diarrhea?

A Salmonella enteritidis

B Clostridum perfringens

C Yersinia enterocolitica

D Shigella flexneri

24.81% People got this right

Explanation:

Correct Answer (B)

Clostridum perfringens

Ref: Read the text below

Sol:

https://emedicoz.com/testresult/683510s14692547 80/189
6/9/24, 12:45 PM Neet PG Preparation, Neet PG Coaching, FMGE, USMLE

Clostridium perfringens causes a toxin mediated non inflammatory diarrhea ,


while all the other three organisms causes invasive inflammatory diarrhea

Question: 83
In patients with hypertrophic cardiomyopathy maximum mutations are
found in which gene:

A b - myosin heavy chain

B Elastin

C a – tropomyosin

D Troponin T

48.77% People got this right

Explanation:

Correct Answer (A)

b - myosin heavy chain

Ref: Read the text below

https://emedicoz.com/testresult/683510s14692547 81/189
6/9/24, 12:45 PM Neet PG Preparation, Neet PG Coaching, FMGE, USMLE

Sol :

Mutations in gene for b - Myosinn heavy chain are associated with 40% of the
families with hypertrophic cardiomyopathy.

• Troponin T mutations - 15% of the families

• α - tropomyosin mutations ~5% of the families

Question: 84
Giant "a" waves in the jugular vein can occur all of following except:

A Constrictive pericarditis.

B Pulmonary hypertension.

C Aortic regurgitation.

D Tricuspid stenosis.

44.69% People got this right

Explanation:

Correct Answer (C)

Aortic regurgitation.

Ref: Read the text below

Sol :

• "a" waves are the positive deflection in the jugular venous pulse following
right atrial contraction.

https://emedicoz.com/testresult/683510s14692547 82/189
6/9/24, 12:45 PM Neet PG Preparation, Neet PG Coaching, FMGE, USMLE

• This becomes giant in situations where the atrium is hypertrophied or


contracts against resistance, such as constrictive pericarditis, pulmonary
hypertension, and tricuspid stenosis or atresia

Question: 85
Most common cause of pleural effusion is

A Trauma

B Left ventricular failure

C Pleural malignancy

D Postoperative
haemorrhage

37.97% People got this right

Explanation:

Correct Answer (B)

Left ventricular failure

Ref: Read the text below

Sol:

• Common causes of transudative effusions are left ventricular heart


failure(Most common cause overall) cirrhosis, and nephrotic syndrome.

• Common causes of exudative effusions are bacterial pneumonia, malignancy,


viral infection, and pulmonary embolism

https://emedicoz.com/testresult/683510s14692547 83/189
6/9/24, 12:45 PM Neet PG Preparation, Neet PG Coaching, FMGE, USMLE

Question: 86
A 24-year-old Primigravida was admitted with fever with chills and right
hypochondriac pain. Hemogram revealed WBC count of 25600 with a
normal platelet count. Her LFT and RFT were normal. USG revealed the
presence of multiple hepatic abscesses. She was started on IV antibiotics.
She developed sudden onset palpitations and her Pulse was rapid and
irregular. BP was 96/64 mm of Hg and she appeared restless. She was
transferred to the ICU and ECG was recorded.

A Sinus Tachycardia

B Paroxysmal
supraventricular tachycardia

C Multifocal Atrial
tachycardia

D Atrial fibrillation

29.28% People got this right

Explanation:

Correct Answer (C)

Multifocal atrial tachycardia

https://emedicoz.com/testresult/683510s14692547 84/189
6/9/24, 12:45 PM Neet PG Preparation, Neet PG Coaching, FMGE, USMLE

MAT is common among ICU patients with severe sepsis. Increased levels of pro-
inflammatory cytokines in the blood are responsible for hyperactivity of the
pacemaker cells leading to this tachyarrhythmia. MAT alone never causes
hypotension and the treatment is only of the underlying cause. On ECG, there is
irregular heart rate with narrow QRS complex tachycardia which can mimic Atrial
fibrillation. However, presence of >=3 different p-waves is diagnostic.

Question: 87
A 20-year-old male presented to the clinic with history of fever with chills
for past 2 days. He complained of associated headache and myalgia. The
highest recorded temperature at home was 41 ֯ C and there was no
particular pattern. Patient is case of Hereditary Spherocytosis and
underwent Splenectomy for the same at 12 years of age since he was
transfusion dependant. His vaccination history is complete. On examination
he had a Temperature of 40 ֯ C, P-90/min, BP-100/60mm of Hg. He had
pallor and icterus with no lymphadenopathy.

Laboratory tests revealed a Hb-7.3gm/dl, WBC-4600, Plt-94000. His Liver


function test showed an elevated bilirubin with elevated ALP, SGOT, SGPT.
Urine analysis showed haemoglobinuria and proteinuria. Examination of
Giemsa-stained thin smear revealed ameboid trophozoites lacking
hemozoin deposits. What is the diagnosis?

A Complicated Malaria

B Leishmaniasis

C Babesiosis

D Meningococcal
septicaemia

25.21% People got this right

https://emedicoz.com/testresult/683510s14692547 85/189
6/9/24, 12:45 PM Neet PG Preparation, Neet PG Coaching, FMGE, USMLE

Explanation:

Correct Answer (C)

Babesiosis

Babesiosis is a malaria-like parasitic disease transmitted by tick bites (most


commonly) or after receiving blood products from an infected donor. Half of
healthy adults with Babesia infection are asymptomatic. When people do
develop symptoms, the most common are fever and hemolytic anemia,
symptoms that are similar to those of malaria. In more severe cases, symptoms
similar to malaria occur, with fevers up to 40.5 °C (105 °F), shaking chills, and
severe anemia (hemolytic anemia). Organ failure may follow, including adult
respiratory distress syndrome. Sepsis in people who have had a splenectomy can
occur rapidly, consistent with overwhelming post-splenectomy infection. Severe
cases are also more likely to occur in the very young, very old, and persons with
immunodeficiency, such as HIV/AIDS patients.

The smear shows hemozoin deficient trophozoites and is diagnostic. In mild-to-


moderate babesiosis, the treatment of choice is a combination of atovaquone
and azithromycin. The standard course is 7 to 10 days, but this is extended to at
least 6 weeks in people with relapsing disease. In severe babesiosis, the
combination of clindamycin and quinine is preferred. In life-threatening cases,
exchange transfusion is performed.

Question: 88
A 70-year-old female presents with complaints of severe unilateral
headache which is worse on supine position. She also reports myalgia,
low grade fever and weight loss of 8 kgs in the last 6 months. There is
history of Jaw claudication with intermittent diplopia. Following
statements about her condition are true EXCEPT

A She is most probably


suffering from a Granulomatous
vasculitis.

https://emedicoz.com/testresult/683510s14692547 86/189
6/9/24, 12:45 PM Neet PG Preparation, Neet PG Coaching, FMGE, USMLE

B Biopsy is likely to reveal


oedematous infiltration of the
intima, media and adventitia with
lymphocytes and macrophages
with giant cells.

C The EULAR
recommendations allow Doppler
study as an alternative to Biopsy
in such cases

D The dreaded complication


in such cases is blindness which is
most commonly due to Anterior
ciliary artery involvement leading
to A.I.O.N.

22.95% People got this right

Explanation:

Correct Answer (D)

The dreaded complication in such cases is blindness which is most commonly


due to Anterior ciliary artery involvement leading to A.I.O.N.

GCA (Giant cell arteritis) is a Large vessel vasculitis which is T-cell mediated i.e
granulomatous.

It typically affects post-menopausal females and throbbing headache is most


common manifestation. Polymyalgia rheumatica is associated in 60-70% of the
cases. The dreaded complication in such cases is blindness which is most
commonly due to Posterior ciliary artery involvement leading to A.I.O.N.
Temporal artery biopsy is the gold standard which reveals oedematous
infiltration of the intima, media and adventitia with lymphocytes and
macrophages with giant cells.

The EULAR recommendations now allow Doppler study of the Superficial


temporal artery as an alternative to Biopsy in such cases. Doppler studies are
likely to reveal “Halo sign”.

https://emedicoz.com/testresult/683510s14692547 87/189
6/9/24, 12:45 PM Neet PG Preparation, Neet PG Coaching, FMGE, USMLE

Question: 89
34-year-old female presented with frequent headaches and bodyache
since the past 4-5 months. She appears stressed and accepts that there is
work pressure on her to meet deadlines. She also complains of inability to
focus on her work. She has disturbed sleep and wakes up tired on
majority of the days. Which antibody is most likely associated with her
condition?

A Anti-Jo

B Anti- Ro

C Anti- Sm

D Anti-polymer

15.11% People got this right

Explanation:

Correct Answer (D)

Anti-polymer antibody

The case described is most likely suffering from Fibromyalgia. She has a classical
triad of aches and pains > 3 months (Pain processing defect) alongwith
disturbed attention span (cognitive defect) and disturbed sleep (NREM defect).
Anti-polymer antibody is found in 50-60% of these patients.

Question: 90
76 years gentleman came to the clinic for a routine follow-up. He is an
obese patient with history of hypertension for 20 years. His BP is controlled
on Telmisartan + Hydrochlorthiazide combination. He is non diabetic and
takes Atorvastatin for his dyslipidemia. He complains of occasional

https://emedicoz.com/testresult/683510s14692547 88/189
6/9/24, 12:45 PM Neet PG Preparation, Neet PG Coaching, FMGE, USMLE

palpitations and feels tired. He lost his wife a month ago and acknowledged
feeling low. ECG was recorded. What is the diagnosis and its probable cause
in this case?

A Ventricular premature
complex – Age related

B Right bundle branch block


– Degenerative HD

C Left bundle branch block –


Ischemic heart disease

D Left ventricular
hypertrophy – hypertensive heart
disease

28.54% People got this right

Explanation:

Correct Answer (B)

Right bundle branch block – Degenerative HD

The symptoms in the case are vague and not specific of any particular disease.
The elderly age, male gender, hypertension constitute risk factors for
development of degenerative HD, Hypertensive HD and IHD. However, the ECG

https://emedicoz.com/testresult/683510s14692547 89/189
6/9/24, 12:45 PM Neet PG Preparation, Neet PG Coaching, FMGE, USMLE

shows rSR’ pattern in lead V1 and confirms a diagnosis of RBBB which is most
probably age related in this case.

LBBB presents with ‘w-shaped’ qrs complex in V1 and indicates IHD

LBBB may also be a sign of LVH.

Question: 91
A 10-year-old boy presents with short stature to the clinic for evaluation.
The boy appears weak and complains of bone pains which did no
respond to Calcium and D3 supplementation. He has history of reccurent
loin pain and was detected to have nephrocalcinosis on USG and started
on Citrate supplementation 6 months ago. What is the most likely
laboratory abnormality in this case?

A High Anion gap metabolic


acidosis

B Elevated Urinary Chloride


level

C Urinary ph > 5.5

D Low Urinary Anion gap

22.79% People got this right

Explanation:

Correct Answer (C)

Urinary ph > 5.5

The case is of Type 1 RTA. In Type 1 RTA(Distal RTA), there is defect in the H+-K+
ATPase pump. It the most common inherited RTA. It is the most severe form of
RTA presenting in young males as Bone disease manifesting as Rickets and Short
stature.

https://emedicoz.com/testresult/683510s14692547 90/189
6/9/24, 12:45 PM Neet PG Preparation, Neet PG Coaching, FMGE, USMLE

It is associated with defect in paracellular Calcium and Magnesium leading to


Hypercalciuria and HypoMg++. This associated with Hyocitraturia increases risk
of Nephrocalcinosis.

ABG shows Metabolic Acidosis. Inspite of acidemia, the urine pH remains > 5.5
due to severe defect in acidification defect in urine.

Due to acidification defect, urinary chloride levels are low and Urinary Anion gap
is high/positive

Question: 92
Fanconi’s triad associated with Proximal RTA consists of all of the
following EXCEPT

A Glycosuria

B Aminoaciduria

C Calciuria

D Phosphaturia

40.29% People got this right

Explanation:

Correct Answer (C)

Calciuria

In Proximal RTA, there is a HCO3 reabsorption defect leading to Metabolic


acidosis. Since proximal tubule is also the site for reabsorption of Glucose,
aminoacid, phosphate and water, the Proximal RTA is commonly associated with
Glycosuria, Aminoaciduria and phosphaturia which is referred to as Fanconi’s
triad.

https://emedicoz.com/testresult/683510s14692547 91/189
6/9/24, 12:45 PM Neet PG Preparation, Neet PG Coaching, FMGE, USMLE

Question: 93
Which one of the following is a Criteria for diagnosis of AKI?

A Urine output <0.5ml/kg/hr


> 12 consecutive hours

B Urine output <0.3ml/kg/hr


> 12 consecutive hours

C Urine output <0.5ml/kg/hr


> 6 consecutive hours

D Urine output <0.3ml/kg/hr


> 6 consecutive hours

31.96% People got this right

Explanation:

Correct Answer (C)

AKI can be diagnosed using KDIGO guidelines.

1. Urine output <=0.5ml/kg/hr > 6 consecutive hours

2. Rise in S.Creatinine > 0.3mg/dl from baseline < 48hours

3. Rise in S.Creatinine > 1.5*Baseline< 7 days

Question: 94
Following statement about Raynaud’s phenomenon are true EXCEPT:

https://emedicoz.com/testresult/683510s14692547 92/189
6/9/24, 12:45 PM Neet PG Preparation, Neet PG Coaching, FMGE, USMLE

A Majority cases are


Idiopathic

B It follows the skin


manifestations in a case of
systemic sclerosis

C Palliative treatment with


Calcium channel blockers is
effective

D Iloprost is approved for its


treatment

26.93% People got this right

Explanation:

Correct Answer (B)

It follows the skin manifestations in a case of systemic sclerosis

In Systemic sclerosis, Raynaud’s precedes the skin manifestations be almost a


decade (i.e. 10 years earlier than the skin manifestations)

Question: 95
Insulin- independent glucose uptake occurs in which of the following
sites?

A Adipose tissue

B Skeletal muscle

C Brain

https://emedicoz.com/testresult/683510s14692547 93/189
6/9/24, 12:45 PM Neet PG Preparation, Neet PG Coaching, FMGE, USMLE

D Cardiac muscle

38.73% People got this right

Explanation:

Correct Answer (C)

Brain

Insulin increases glucose uptake occurs in skeletal muscle, cardiac muscle,


smooth muscle, adipose tissue, leukocytes and liver. In most insulin-sensitive
tissues, glucose increases glucose transport by enhancing the facilitated
diffusion of glucose down a concentration gradient. In the liver, where glucose
freely permeates the cell membrane, glucose uptake is increased because of
phosphorylation by glucokinase. Formation of gloucose-6-phosphate reduces
the intracellular concentration of free glucose and maintains the concentration
gradient favouring movement of glucose into the cell.

Question: 96
A 58-year-old man is hospitalized following an acute myocardial
infarction. Several days later, the patient’s 24-hour urine output is lower
than normal. An increase in which of the following contributes to a
reduced urine flow in a patient with CCF?

A Atrial natriuretic peptide

B Renal perfusion pressure

C Renal sympathetic nerve


activity

D Renal natriuretic peptide

28.19% People got this right

https://emedicoz.com/testresult/683510s14692547 94/189
6/9/24, 12:45 PM Neet PG Preparation, Neet PG Coaching, FMGE, USMLE

Explanation:

Correct Answer (C)

Renal sympathetic nerve activity

Patients with CCF have a paradoxical increase in sodium and water retention
despite an increase in ECF volume. An increase in renal sympathetic nerve
activity causes decrease in NaCl and water excretion by decreasing GFR,
increasing renin secretion and increasing tubular NaCl reabsorption.

Increase in ANP, increased renal perfusion pressure, increased renal natriuretic


peptide (urodilantin) increase sodium and water excretion.

Question: 97
Dietary fat, after being processed, is extruded from the mucosal cells of
the GI tract into the lymphatic ducts in which of the following forms?

A Diglycerides

B Monoglycerides

C Free fatty acids

D Chylomicrons

55.66% People got this right

Explanation:

Correct Answer (D)

Chylomicrons

Triglycerides are hydrolyzed to 2-monoglyceride and free fatty acids, which enter
into the mucosal cells. In the mucosal cells, re-esterification of monoglycerides,
long chain fatty acids, cholesterol, fat soluble vitamins results in the formation of

https://emedicoz.com/testresult/683510s14692547 95/189
6/9/24, 12:45 PM Neet PG Preparation, Neet PG Coaching, FMGE, USMLE

chylomicrons which enter into the lymphatics. Short chain fatty acids (with less
than 10-12 carbon atoms) are extruded in the form of free fatty acids into the
portal blood.

Question: 98
A 20-year-old man ruptures his spleen in a motorcycle accident. His blood
pressure at the time of admission is 80/60 mm Hg. A decrease in which of
the following is expected in this patient?

A Heart rate

B Myocardial contractility

C Total peripheral resistance

D Venous compliance

29.38% People got this right

Explanation:

Correct Answer (D)

Venous compliance

Hypotension will elicit the baroreceptor reflex which increases the sympathetic
discharge to the cardiovascular system. Increased sympathetic activity results in
increase in heart rate, increase in myocardial contractility and vasoconstriction.
Vasoconstriction increases the total peripheral resistance. Increased sympathetic
activity also produces a venoconstriction which decreases the venous
compliance and increases venous return.

https://emedicoz.com/testresult/683510s14692547 96/189
6/9/24, 12:45 PM Neet PG Preparation, Neet PG Coaching, FMGE, USMLE

Question: 99
Normally intrapleural pressure is negative throughout a tidal inspiration
and expiration because of which of the following?

A Lungs tend to recoil


towards the outside throughout a
tidal breath

B Chest wall tends to recoil


inward throughout a tidal breath

C Lungs and chest wall recoil


away from each other throughout
a tidal breath

D Lungs and chest wall recoil


in the same direction throughout
a tidal breath

54.07% People got this right

Explanation:

Correct Answer (C)

Lungs and chest wall recoil away from each other throughout a tidal breath

When the pleura, and hence the lung- chest wall system, are intact, the inward
recoil of the lung (collapsing force) opposing the outward elastic recoil of the
chest wall (expanding force) results in a subatmospheric (negative) pressure
within the pleural space.

Question: 100
Which of the following receptors is responsible for detecting deep
pressure?

https://emedicoz.com/testresult/683510s14692547 97/189
6/9/24, 12:45 PM Neet PG Preparation, Neet PG Coaching, FMGE, USMLE

A Pacinian corpuscle

B Meissner’s corpuscle

C Merkel’s disc

D Ruffini’s endings

41.62% People got this right

Explanation:

Correct Answer (A)

Pacinian corpuscle

Pacinian corpuscle is a very rapidly adapting touch receptor and is specific for
fast vibration upto 800 cps and for deep pressure (poking).

Meissner’s corpuscle is an elongated, encapsulated, phasic receptor, present in


glabrous skin and especially abundant in fingertips, lips and nipples. It is meant
for detection of fine, well localized touch (also k/a discriminative touch), texture,
topognosis and slow vibration up to 80 cps.

Merkel’s disc is a tonic receptor in hairy and glabrous skin and is meant for fine,
well localized touch.

Ruffini’s ending is a tonic receptor in the dermis of hairy and glabrous skin that
will produce a train of action potentials proportional to the intensity of pressure
applied to the skin. Ruffini’s endings respond to stretch, sustained pressure,
fluttering vibration and touch.

Question: 101
Which of the following is Primary prevention measure:

https://emedicoz.com/testresult/683510s14692547 98/189
6/9/24, 12:45 PM Neet PG Preparation, Neet PG Coaching, FMGE, USMLE

A Source reduction

B MDT for leprosy

C Breast self examination

D Physiotherapy in polio

38.66% People got this right

Explanation:

Correct Answer (A)

a. Source reduction : Primary prevention

b. MDT for leprosy : Secondary prevention

c. Breast self examination : Secondary prevention

d. Physiotherapy in polio: Tertiary prevention

Question: 102
Pearson coefficient between salt intake and blood pressure is 0.7 Which
of the following is true

A It indicates obesity is a risk


factor for hypertension

B 49% of variation in BP is
influenced by salt intake

C It indicates negative
correlation

https://emedicoz.com/testresult/683510s14692547 99/189
6/9/24, 12:45 PM Neet PG Preparation, Neet PG Coaching, FMGE, USMLE

D All of these

30.17% People got this right

Explanation:

Correct Answer (B)

• Corelation does not prove causation or risk

• Coefficient of determination : 0.7 x 0.7 = 0.49 . So 49% of variation in BP is


influenced by salt intake

• It indicates positive correlation

Question: 103
Which of the following is mortality indicator :

A Incidence

B Life expectancy

C Prevalence

D DALYs

38.36% People got this right

Explanation:

Correct Answer (B)

• Incidence : Morbidity indicator

• Life expectancy : Mortality indicator

• Prevalence : Morbidity indicator

https://emedicoz.com/testresult/683510s14692547 100/189
6/9/24, 12:45 PM Neet PG Preparation, Neet PG Coaching, FMGE, USMLE

• DALYs : disability indicator

Question: 104
Out of cases 60% are exposed and out of controls 80% are non exposed.
Which of the following is true :

A Positive association

B Negative association

C No association

D Study design is not


sufficient to comment on
association

37.01% People got this right

Explanation:

Correct Answer (A)

Question: 105

https://emedicoz.com/testresult/683510s14692547 101/189
6/9/24, 12:45 PM Neet PG Preparation, Neet PG Coaching, FMGE, USMLE

A new treatment can reduce mortality in cancer patients from 20% to 10%
. Calculate Relative risk reduction:

A 5%

B 10%

C 25%

D 50%

33.92% People got this right

Explanation:

Correct Answer (D)

RRR= 20-10 / 20 = 50%

Question: 106
Which of the following is true for the table shown below :

A Sensitivity is same as
Specificity

https://emedicoz.com/testresult/683510s14692547 102/189
6/9/24, 12:45 PM Neet PG Preparation, Neet PG Coaching, FMGE, USMLE

B Sensitivity is more than


Specificity

C Sensitivity is less than


Specificity

D None of these

47.16% People got this right

Explanation:

Correct Answer (C)

• Sensitivity = 400 / 500 = 80%

• Specificity = 450 / 500 = 90%

Question: 107
Which of the following adverse effect of vaccines is correctly matched :

A SA 14 14 2 : Vaccine
associated viscerotropic disease

B Measles : Brachial neuritis

C Rotavac: Guillain-Barre
syndrome

D None of these

13.2% People got this right

Explanation:

Correct Answer (D)

https://emedicoz.com/testresult/683510s14692547 103/189
6/9/24, 12:45 PM Neet PG Preparation, Neet PG Coaching, FMGE, USMLE

• Vaccine associated viscerotropic disease : 17 D

• Brachial neuritis : TT

• Guillain-Barre syndrome : Killed influenza vaccine

Question: 108
All of the following indicators are included under Multidimensional
poverty index except :

A Child mortality

B Maternal mortality

C School attendance

D Safe drinking water

30.42% People got this right

Explanation:

Correct Answer (B)

https://emedicoz.com/testresult/683510s14692547 104/189
6/9/24, 12:45 PM Neet PG Preparation, Neet PG Coaching, FMGE, USMLE

Question: 109
Which of the following is/are disposed in red bag during covid
management :

A Used mask

B Head cover

C Throat swab

D Goggles

41.71% People got this right

Explanation:

Correct Answer (D)

o Used mask : Yellow bag

o Head cover : Yellow bag

o Throat swab : Yellow bag

o Goggles: Red bag

Question: 110
Which of the following is true for image shown below :

https://emedicoz.com/testresult/683510s14692547 105/189
6/9/24, 12:45 PM Neet PG Preparation, Neet PG Coaching, FMGE, USMLE

A It's under NIS in all states

B Killed vaccine used in India

C Given subcutaneous route


under schedule

D Diluted with normal saline

47.07% People got this right

Explanation:

Correct Answer (C)

• It's under NIS in endemic states

• Live vaccine used in India

• Given subcutaneous route under schedule

• Diluted with PBS

Question: 111
A study is done with the help of existing records to save time and cost .
Risk ratio obtained was 1.8. Positive association was seen between arsenic
exposure and mesothelioma among factory workers . Study design is :
https://emedicoz.com/testresult/683510s14692547 106/189
6/9/24, 12:45 PM Neet PG Preparation, Neet PG Coaching, FMGE, USMLE

A Prospective cohort study

B Retrospective cohort study

C Case control study

D Cross sectional study

48.88% People got this right

Explanation:

Correct Answer (B)

Retrospective cohort study : is done with the help of existing records to save
time and cost . Aka historical cohort study

Question: 112
The error of rejecting a true null hypothesis is :

A Type I error

B Type II error

C Both of these

D None of these

41.29% People got this right

Explanation:

Correct Answer (A)

https://emedicoz.com/testresult/683510s14692547 107/189
6/9/24, 12:45 PM Neet PG Preparation, Neet PG Coaching, FMGE, USMLE

The error of rejecting a true null hypothesis is : Type I error Aka false positive
error

Question: 113
An anganwadi teacher takes weight and height of a 4 year old and finds
out that height for age < -2 SD . Likely finding is :

A Chronic malnutrition

B Acute malnutrition

C Recent malnutrition

D No malnutrition

49.79% People got this right

Explanation:

Correct Answer (A)

Expln :

Question: 114
https://emedicoz.com/testresult/683510s14692547 108/189
6/9/24, 12:45 PM Neet PG Preparation, Neet PG Coaching, FMGE, USMLE

A case of RTA was presenting to Emergency room. Disinfectant used for


blood spill management on hospital floor :

A Ethyl alchohol

B Chlorhexidine

C Formaldehyde

D Sodium hypochlorite

53.96% People got this right

Explanation:

Correct Answer (D)

Expln :

0.5% Sodium hypochlorite is used for spill management

Question: 115
A teacher taught his student handwashing . Student learns it and do
perform it at home and schools. What is this mechanism of learning :

A Affective and cognitive

B Cognitive

C Psychomotor

D Affective

https://emedicoz.com/testresult/683510s14692547 109/189
6/9/24, 12:45 PM Neet PG Preparation, Neet PG Coaching, FMGE, USMLE

24.81% People got this right

Explanation:

Correct Answer (C)

Expln :

• Learning - any relative permanent change in behavior.

3 domain – Cognitive learning (knowledge)

Affective learning (Attitude)

Psychomotor learning (skills/Practices)

Question: 116
Decreasing order of mortality in neonates all over the world from maximum
to minimum

i) Congenital anomalies

ii) Prematurity and LBW

iii) Sepsis

iv) Birth asphyxia

A i,ii,iii,iv

B iv,iii,ii,i

C ii, iii, iv, i

D ii, iv, iii, i

33.03% People got this right

Explanation:

https://emedicoz.com/testresult/683510s14692547 110/189
6/9/24, 12:45 PM Neet PG Preparation, Neet PG Coaching, FMGE, USMLE

Correct Answer (D)

Question: 117
Arrange the mortality in children between 1-59 months from maximum to
minimum

i) Pneumonia

ii) Diarrhoea

iii) Malaria

iv) Injury

A i,ii,iii,iv

B i,ii,iv,iii

C iv,iii,ii,i

D iv,i,iii,ii

34.24% People got this right

Explanation:

Correct Answer (B)

https://emedicoz.com/testresult/683510s14692547 111/189
6/9/24, 12:45 PM Neet PG Preparation, Neet PG Coaching, FMGE, USMLE

Question: 118
Height for age<-2SD Likely cause?

A Recent Infection

B Acute Malnutrition

C Chronic Malnutrition

D No malnutrition

57.05% People got this right

Explanation:

Correct Answer (C)

Height for age is a marker of stunting or chronic malnutrition

Question: 119
A 12 years old has history of cyanosis. His X Ray is shown.

https://emedicoz.com/testresult/683510s14692547 112/189
6/9/24, 12:45 PM Neet PG Preparation, Neet PG Coaching, FMGE, USMLE

What is most likely diagnosis?

A Snowman heart-TAPVC

B Boot shaped-TOF

C Egg on side- dTGA

D Cottage loaf- Truncus


arteriosus

63.59% People got this right

Explanation:

Correct Answer (A)

Snowman heart- TAPVC

Question: 120
Which of the following is not a cause of cyanosis at birth?

A dTGA

https://emedicoz.com/testresult/683510s14692547 113/189
6/9/24, 12:45 PM Neet PG Preparation, Neet PG Coaching, FMGE, USMLE

B TAPVC

C TOF

D Tricuspid atresia

28.19% People got this right

Explanation:

Correct Answer (C)

TOF usually presents after the first year of life

Question: 121
An expectant mother is seen for a prenatal visit. Prenatal genetic testing
performed on her fetus showed karyotype 45,X. She will be meeting with a
genetic counselor in the coming days.

Of the following, the MOST likely cardiac disorder to be seen in this fetus is

A Aortopulmonary window

B Coarctation of the aorta

C Tetralogy of Fallot

D Truncus arteriosus

53.77% People got this right

Explanation:

Correct Answer (B)

https://emedicoz.com/testresult/683510s14692547 114/189
6/9/24, 12:45 PM Neet PG Preparation, Neet PG Coaching, FMGE, USMLE

A karyotype of 45,X, is consistent with Turner syndrome (TS). Patients with TS are
known to have an increased incidence of left-sided cardiac lesions consisting of
bicuspid aortic valve and coarctation of the aorta. Other vascular anomalies that
have been noted in TS include anomalous pulmonary venous connection and
persistent left superior vena cava. Dilation of the aorta in general, as well as
dilation of other vessels such as the brachial and carotid arteries, may also be
seen. Tetralogy of Fallot, truncus arteriosus, and aortopulmonary window have
not been associated with TS. Affected patients can have resting sinus tachycardia
and prolonged QT syndrome.

Question: 122
A 6-month-old male infant who is limp and exhibiting obtundation in the
morning is brought to the emergency department. Bedside blood analysis
reveals a significant metabolic acidosis, including an elevated lactate level
and a capillary blood glucose level of 35 mg/dL (1.9 mmol/L) and 3+
ketones in the urine. His newborn screening results were normal. His
parents report that he only recently has been able to sleep through the
night without waking for a feeding and that he has had a few episodes of
morning lethargy that improved after his first morning feeding. The infant
appears small and nearly comatose. He has chubby cheeks, a short nose,
and a relatively small chin. He has a temperature of 37.8°C, a heart rate of
155 beats/min, a respiratory rate of 15 breaths/min, and a blood pressure of
75/45 mm Hg. Cardiac examination reveals no murmur and a normal S1 and
S2. The liver edge is palpable 3 cm below the right costal margin. The
remainder of the physical examination findings are unremarkable.

Of the following, the MOST likely diagnosis is

A Fatty acid oxidation


disorder

B Glycogen storage disorder

https://emedicoz.com/testresult/683510s14692547 115/189
6/9/24, 12:45 PM Neet PG Preparation, Neet PG Coaching, FMGE, USMLE

C Mitochondrial disorder

D Organic acid disorder

42.76% People got this right

Explanation:

Correct Answer (B)

The infant in the vignette, with obtundation after prolonged overnight fasting
and hypoglycemia with lactic acidosis and ketosis, most likely has a glycogen
storage disorder (GSD). Glycogen storage disease type 1a (also known as von
Gierke disease) is caused by a deficiency in glucose-6-phosphatase. This type
often presents in infancy and results in hepatomegaly, growth failure, and
recurrent episodes of hypoglycemia with ketosis. Although some neonates with
glycogen storage disease present with severe hypoglycemia, it is more common
for infants aged 3 to 4 months to manifest hepatomegaly, lactic acidosis,
hypoglycemic seizures, or a combination of these. Additional laboratory
abnormalities include elevated uric acid levels and hyperlipidemia. The physical
findings of chubby cheeks, a short nose, and a relatively small chin are
nonspecific but can be found in children with GSD. The diagnosis is made by
means of genetic analysis identifying pathologic variants in specific genes,
including G6PC, SLC37A4, or both. These genes encode for glucose-6-
phosphatase activity and glucose-6-phosphate exchanger SLC37A4 activity,
respectively.

Question: 123
A female neonate born at term is evaluated in the neonatal intensive care
unit. The pregnancy was complicated by maternal gestational diabetes and
alcohol exposure. Prenatal ultrasonography at 20 weeks showed
intrauterine growth retardation. The neonate is presently intubated. Her
birth weight is 2,200 g. She has a small jaw, clenched hands with
overlapping fingers, rocker-bottom feet, a short sternum, and hypoplastic
nails. Echocardiography shows a ventricular septal defect. Renal
ultrasonography shows a horseshoe kidney.
https://emedicoz.com/testresult/683510s14692547 116/189
6/9/24, 12:45 PM Neet PG Preparation, Neet PG Coaching, FMGE, USMLE

Of the following, the MOST likely diagnosis is

A Fetal alcohol spectrum


disorder

B Trisomy 13

C Trisomy 18

D Trisomy 21

34.22% People got this right

Explanation:

Correct Answer (C)

The neonate in the vignette has trisomy 18. Trisomy 18 is characterized by


intrauterine growth retardation and by congenital cardiac, ocular, and renal
anomalies. Dysmorphic features include small jaw, clenched hands with
overlapping fingers (index overlapping the third, fifth overlapping the fourth),
short sternum, and hypoplastic nails. The majority of pregnancies affected by
trisomy 18 result in fetal loss during the first trimester. Infants who make it to
term have a high rate of infant mortality, with one-half not surviving 1 week and
only 5% to 10% living beyond the first year after birth. Care of neonates with
trisomy 18 involves a multidisciplinary approach with emphasis on growth and
feeding, development, respiratory requirements, and specialty-specific care
according to the congenital anomalies. The most frequent causes of death

https://emedicoz.com/testresult/683510s14692547 117/189
6/9/24, 12:45 PM Neet PG Preparation, Neet PG Coaching, FMGE, USMLE

include central apnea, complications of cardiac malformations, and respiratory


failure related to obstruction and aspiration.

Question: 124
A pediatrician receives a phone call from the state’s newborn screening
program regarding a 7-day-old neonate with an elevated trypsinogen,
which is concerning for a diagnosis of cystic fibrosis. The neonate was seen
by the pediatrician’s partner 1 day ago and had appropriate feeding and
growth parameters as well as normal vital signs and physical examination
findings. The partner noted that there is no significant family history of
diseases in childhood.

Of the following, the MOST appropriate next step is to

A Inform the parents of the


diagnosis of cystic fibrosis

B Discuss the result at the 2-


month health supervision visit

C Reassure the parents that


this is likely a false-positive result

D Schedule the neonate for


diagnostic testing

44.18% People got this right

Explanation:

Correct Answer (D)

This infant’s newborn screening test resulted in an elevated immunoreactive


trypsinogen (IRT), a pancreatic enzyme that is elevated in most newborns with

https://emedicoz.com/testresult/683510s14692547 118/189
6/9/24, 12:45 PM Neet PG Preparation, Neet PG Coaching, FMGE, USMLE

cystic fibrosis (CF). An elevated IRT is a sensitive but not a specific screen for CF;
further diagnostic testing with a sweat chloride test is necessary. It would,
therefore, not be appropriate to notify the neonate’s parents of a diagnosis at
this time. Early identification and treatment of CF has been associated with
maximal growth and improved lung function; it would not be appropriate to wait
for the 2-month health supervision visit to address this laboratory result.
Although false-positive IRT elevations can occur, especially in premature or ill
neonates, follow-up testing is necessary before any result can be labeled a false
positive. The most appropriate next step is to schedule diagnostic testing with a
sweat chloride test.

Question: 125
What is the screening test of choice for metabolic disorders and inborn
errors of metabolism?

A TSH

B Urine GCMS

C Blood TMS

D Whole exome sequencing

45.23% People got this right

Explanation:

Correct Answer (C)

Blood TMS

Question: 126

https://emedicoz.com/testresult/683510s14692547 119/189
6/9/24, 12:45 PM Neet PG Preparation, Neet PG Coaching, FMGE, USMLE

A 33-year-old woman presents with a diffuse scaly skin rash of 4 weeks


duration. Biopsy of lesional skin reveals mycosis fungoides. Which of the
following IHC markers would be most useful for identifying malignant
cells in the skin of this patient?

A Calcitonin

B CD4

C Desmin

D HMB-45

29.05% People got this right

Explanation:

Correct Answer (B)

CD4. Mycosis Fungoides is a T cell lymphoma. CD4 is a T cell marker. Calcitonin


is used in medullary carcinoma of thyroid, Desmin in Muscle tumors and HMB 45
is used for melanoma.

Question: 127
A 6 year child presents with failure to thrive, diarrhea and steatorrhea for
last 5 years. Peripheral blood shows acanthocytes. Stool examination
confirms fat in the stools. Likely diagnosis is?

A Whipple Disease

B Celiac Disease

C Abetalipoproteinemia

https://emedicoz.com/testresult/683510s14692547 120/189
6/9/24, 12:45 PM Neet PG Preparation, Neet PG Coaching, FMGE, USMLE

D Tropical sprue

42.85% People got this right

Explanation:

Correct Answer (C)

Abetalipoproteinemia. Acanthocytes in PBS is the clue seen in


Abetalipoproteinemia. Which occurs due to MTP mutation.

Question: 128
A 35 year old is having long standing diarrhea with multiple
hyperpigmented lesion in skin. Blood glucose is 250mg/dL. Which of the
following is not true about the disease?

A Type I diabetes is cause

B Rarely can cause cirrhosis

C Testicular atrophy can be


seen

D Arthritis can be
manifestation.

11.73% People got this right

Explanation:

Correct Answer (B)

The patient is suspected to have hemochromatosis ( diarrhoea, skin


hyperpigmentation and hyperglycemia) in Hemochromatosis Cirrhosis is one of
the most common manifestation.

https://emedicoz.com/testresult/683510s14692547 121/189
6/9/24, 12:45 PM Neet PG Preparation, Neet PG Coaching, FMGE, USMLE

Question: 129
A 25-year-old G2P1 came to OPD with an ulcer in the oral cavity as shown
below. The lesion was bit ulcerated as shown in the figure below. The
physician suspected a diagnosis and sent for biopsy which showed –
Multiple tiny capillaries resembling granulation tissue. What is the likely
diagnosis?

A Squamous cell carcinoma

B Pyogenic granuloma

C Infectious ulcer

D Cavernous hemangioma

26.33% People got this right

Explanation:

Correct Answer (B)

Pyogenic Granuloma.

The pyogenic granuloma

• Its’ an inflammatory lesion typically found on the gingiva of children, young


adults, and pregnant women (pregnancy tumor).

https://emedicoz.com/testresult/683510s14692547 122/189
6/9/24, 12:45 PM Neet PG Preparation, Neet PG Coaching, FMGE, USMLE

• Also known as lobular capillary hemangioma

• The surface of the lesion is often ulcerated and red to purple in color

• Histologically these lesions demonstrate a highly vascular proliferation of


organizing granulation tissue.

• Pyogenic granulomas can regress, mature into dense fibrous masses

The clinical history of pregnancy and the classic location and appearance are
clues.

Question: 130
A 25 year old male presented with a 2cm thyoid nodule. A throidectomy
was done. The histology picture is given below. Which of the following
genetic change is seen in the below condition?

A BRAF mutation

B RAS Mutation

C Menin mutation

D APC mutation.

33.87% People got this right

https://emedicoz.com/testresult/683510s14692547 123/189
6/9/24, 12:45 PM Neet PG Preparation, Neet PG Coaching, FMGE, USMLE

Explanation:

Correct Answer (A)

Diagnosis is Papillary carcinoma thyroid.

Genetics of Thyroid neoplasms.

• Most papillary carcinomas have gain-of-function mutations involving the


genes encoding the RET or NTRK1 receptor tyrosine kinases, or in the
serine/threonine kinase BRAF

• Follicular carcinomas are associated with acquired muta- tions that activate
RAS or the PI-3K/AKT arm of the receptor tyrosine kinase signaling pathway

• Medullary carcinoma thyroid – RET mutations

Question: 131
Maccullum plaques are seen in

A Infective endocarditis

B Atherosclerosis of aorta

C Aortic dissection

D Rheumatic heart disease

28.79% People got this right

Explanation:

Correct Answer (D)

Mural endocardial lesions can be seen as MacCallum plaques in rheumatic heart


disease.

https://emedicoz.com/testresult/683510s14692547 124/189
6/9/24, 12:45 PM Neet PG Preparation, Neet PG Coaching, FMGE, USMLE

• These plaques appear as map-like areas of thickened, roughened, and


wrinkled part of the endocardium in the left atrium.

• Perhaps they are caused by regurgitant jets of blood flow, due to


incompetence of the mitral valve.

• Microscopy shows diffuse sub-endocardial fibrosis

Question: 132
A 50-year-old with diagnosis of restrictive cardiomyopathy underwent a
biopsy. The histo-pathological picture was as below. The most possible
diagnosis?

A Hypersensitive myocarditis

B Uremic pericarditis

C Amyloidosis of heart

D Hemochromatosis

49.51% People got this right

Explanation:

Correct Answer (C)

https://emedicoz.com/testresult/683510s14692547 125/189
6/9/24, 12:45 PM Neet PG Preparation, Neet PG Coaching, FMGE, USMLE

The picture shows - Hematoxylin and eosin stain, showing amyloid appearing as
amorphous pink material around myocytes. Second picture – Congo red.

Amyloidosis results from the extracellular accumulation of protein fibrils that are
prone to forming insoluble β-pleated sheets

• Senile cardiac amyloidosis characteristically occurs in individuals 70 years and


older, and has a far better prognosis than systemic amyloi- dosis.

• Cardiac amyloidosis most frequently produces a restric- tive cardiomyopathy,


but it can also be asymptomatic, manifest as dilation or arrhythmias, or mimic
ischemic or valvular disease.

• In cardiac amyloidosis the heart varies in consistency from normal to firm


and rubbery.

Histologically, hyaline eosinophilic deposits of amyloid may be found in the


interstitium, conduction tissue, valves, endocardium, pericardium

Question: 133
Which of the following is true regarding the gross picture shown below.

A Dark areas are viable

B Dark areas are fibrotic

C Dark areas are in zone 3

https://emedicoz.com/testresult/683510s14692547 126/189
6/9/24, 12:45 PM Neet PG Preparation, Neet PG Coaching, FMGE, USMLE

D White areas are white


infarcts.

17.48% People got this right

Explanation:

Correct Answer (C)

The image shows CVS of liver (Nutmeg liver) dark areas are hemorrhagic
necrosis seen in Zone 3.

Question: 134
The correct description of the RBC (marked with arrow) in the image and
the condition in which it is seen is?

A Schistiocyte, Thalassemia

B Sickle cell, Sickle cell


anemia

C Dacryocyte, Thalassemia

D Target cell, Thalassemia

47.16% People got this right


https://emedicoz.com/testresult/683510s14692547 127/189
6/9/24, 12:45 PM Neet PG Preparation, Neet PG Coaching, FMGE, USMLE

Explanation:

Correct Answer (C)

Dacryocyte thalassemia. Tear drop RBC or dacryocyte are seen in conditions


where EMH s happening

Question: 135
Match the following

A 1-D, 2-C, 3-A, 4-B

B 1-C, 2-D, 3-B, 4-A

C 1-C, 2-D, 3-A, 4-B

D 1-D, 2-C, 3-B, 4-A

38.55% People got this right

Explanation:

Correct Answer (C)

1-C, 2-D, 3-A, 4-B

https://emedicoz.com/testresult/683510s14692547 128/189
6/9/24, 12:45 PM Neet PG Preparation, Neet PG Coaching, FMGE, USMLE

Question: 136
A 45-year HIV patient presents with fever and night sweats. Examination
shows generalised lymphadenopathy and splenomegaly. Biopsy from one
of the lymphnode shows hyalinisation of follicle and onion skinning as
shown in the image. Likely diagnosis?

A Castel Mann Disease

B Hodgkin Lymphoma

C DLBCL

D Burkitt Lymphoma

25.72% People got this right

Explanation:

Correct Answer (A)

Castelmann Disease is frequently seen in HIV patients an d presents with


generalized lymphadenopathy. Biopsy shows hyalinization of vessels and onion
skinning.

Question: 137
https://emedicoz.com/testresult/683510s14692547 129/189
6/9/24, 12:45 PM Neet PG Preparation, Neet PG Coaching, FMGE, USMLE

A 53-year old presents with epistaxis. Biopsy from the mass shows a
tumor with small round cells centered around the blood vessel.
Immunohistochemistry shows the neoplastic cells are CD45 (LCA)+, CD3-,
CD4- CD20-, CD16+, EBV+. Which of the following is most likely
diagnosis?

A Nasopharyngeal carcinoma

B Extranodal NK cell
lymphoma

C Anaplastic large cell


lymphoma

D Hodgkin lymphoma

18.67% People got this right

Explanation:

Correct Answer (B)

Extranodal NK cell lymphoma. The histologica description is Angiocentric pattern


of lymphoma cells and IHC is positive for CD16 which is a marker of NK cells

Question: 138
True about TRALI are all except

A Occurs with 6 of
transfusion

B Steroids are not Indicated

https://emedicoz.com/testresult/683510s14692547 130/189
6/9/24, 12:45 PM Neet PG Preparation, Neet PG Coaching, FMGE, USMLE

C Occurs due to anti HLA


antibodies in the patient

D Occurs after plasma


products

14.27% People got this right

Explanation:

Correct Answer (C)

Occurs due to anti HLA antibodies in patient. TRALI occurs due to anti HLA
antibodies in donors plasma not in recipient

Question: 139
A 6-year boy presents with multiple petechial patches and purpura after
respiratory tract infection. Platelet count is 40000. Which of the following
is true regarding this case

A Transfuse platelets

B Deficiency of GP IIb-IIIa

C BM will show increased


megakaryocytes

D Type IV hyper sensitivity

29.96% People got this right

Explanation:

Correct Answer (C)

https://emedicoz.com/testresult/683510s14692547 131/189
6/9/24, 12:45 PM Neet PG Preparation, Neet PG Coaching, FMGE, USMLE

BM will show increased megakaryocytes. This is a case of Acute ITP, ITP is


megakaryocytic thrombocytopenia, thus answer C

Question: 140
A 25-year old female presents with cough and hemoptysis, biopsy from
the lung shows casseating granuloma. A TB PCR was positive. Flow
cytometric analysis of T cells isolated from lesion show T helper cells.
Which of the following is correctly matched regarding these helper cells?

A TH1 - IL4

B TH2 - IL4

C TH1 – INF γ

D TH2 – INFγ

28.21% People got this right

Explanation:

Correct Answer (C)

TH1 – INF γ. Granuloma formation requires Th1 cells and they secrete INF γ

Question: 141
All the following cause painful ulcers except:

A Chancroid

B Pyoderma gangrenosum

https://emedicoz.com/testresult/683510s14692547 132/189
6/9/24, 12:45 PM Neet PG Preparation, Neet PG Coaching, FMGE, USMLE

C Tuberculous chancre

D Aphthous ulcer

35.24% People got this right

Explanation:

Correct Answer (C)

Question: 142
An 8 year old child presents with localized alopecia over scalp with scales.
The diagnosis is:

A Alopecia areata

B Trichotillomania

C Tinea capitis

D Lichen planus

45.41% People got this right

Explanation:

Correct Answer (C)

Child age group would support T. capitis diagnosis. Also, localised scaly plaques
(grey patch) would be in T. capitis. Alopecia areata would not be scaly.
Trichotillomania is unusual in children also would not have scaling. Lichen planus
is also not scaly but has scarring localised patches.

https://emedicoz.com/testresult/683510s14692547 133/189
6/9/24, 12:45 PM Neet PG Preparation, Neet PG Coaching, FMGE, USMLE

Question: 143
A 20-year-old patient comes with widespread itchy oozing plaques on the
body including on the back of the knee. He also has a history of recurrent
such lesions. Which drug would not be helpful in this patient.

A Azathioprine

B Phototherapy

C Steroid

D Retinoid

22.02% People got this right

Explanation:

Correct Answer (D)

Oozing and itching are classical signs of atopic eczema. Flexural involvement
(back of the knee) would suggest adult atopic eczema. Immunosupressives
(A,B,C) help the patient. Retinoid dry up the skin and would make eczema worse
as atopic eczema patient have barrier loss (loss of intercellular cement) and
would need moisturisers instead of drying agents.

Question: 144
A patient presents with itchy lesions on trunk as shown. Subsequently, new
lesions are seen on the trunk along the ribs. Pick the true statement:

https://emedicoz.com/testresult/683510s14692547 134/189
6/9/24, 12:45 PM Neet PG Preparation, Neet PG Coaching, FMGE, USMLE

A Topical antifungals need to


be given

B Sensations should be
checked

C Steroids would increase the


disease

D Viral etiology

21.95% People got this right

Explanation:

Correct Answer (D)

Question: 145
Koebner’s phenomenon is not seen in

A Molluscum

B Darier disease

https://emedicoz.com/testresult/683510s14692547 135/189
6/9/24, 12:45 PM Neet PG Preparation, Neet PG Coaching, FMGE, USMLE

C Lichen planus

D Contact leucoderma

35.89% People got this right

Explanation:

Correct Answer (D)

Question: 146
Which of the following investigation does not match with the diagnosis
of iron deficiency anemia in pregnancy?

A Serum ferritin levels of


6mg/litre

B Serum iron levels of


12mcg/dl

C TIBC 260mcg/dl

D Reduced serum hepcidin


levels

12.8% People got this right

Explanation:

Correct Answer (C)

TIBC 260mcg/dl

In iron deficiency anemia, there is reduction in the levels of Sr. iron, Sr. Ferritin,
Sr. Hepcidin MCV, MCHC but there is an increase in the levels of TIBC,
erythropoietin and red cell distribution width.

https://emedicoz.com/testresult/683510s14692547 136/189
6/9/24, 12:45 PM Neet PG Preparation, Neet PG Coaching, FMGE, USMLE

(Reference: Williams obstetrics, Medical disorders, Haematological disorders,


anemia in pregnancy.)

Question: 147
A 26 years old lady presented with acute onset of breathlessness and
fatigue for 5-7 days. She had a full-term normal delivery 1 month back.
She had mild pre-eclampsia during antenatal period but her BP was
normal after delivery. Her Hb is 11.2gm%. There is no history of any cardio
respiratory disease in past. Her chest X ray is normal. Her 2D echo shows
LVEF of 35% with no cardiac lesion. Her ECG is within normal limits.
Which of the following is the most likely cause?

A LRTI

B Pulmonary embolism

C Peripartum
cardiomyopathy

D Postpartum psychosis

51.49% People got this right

Explanation:

Correct Answer (C)

Peripartum cardiomyopathy.

In peripartum cardiomyopathy, there is biventricular dysfunction due to damage


to the cardiac muscles in pregnancy. The LVEF falls <45%, it occurs 1 month
before delivery to 5 months after delivery, there is no other cause of CCF and
there is no antecedent cardiac lesion.

https://emedicoz.com/testresult/683510s14692547 137/189
6/9/24, 12:45 PM Neet PG Preparation, Neet PG Coaching, FMGE, USMLE

(Reference: Williams obstetrics, Medical disorders, CVS DISEASES< peripartum


cardiomyopathy.)

Question: 148
A 32 years old G3A2 has 32 weeks of gestation has had convulsion of
generalised tonic clonic type. She has been brought in emergency room
with post-ictal drowsiness and BP of 160/110mm Hg. Which of the
following is true regarding this case?

A MgSO4 acts as anti-


convulsant of choice and it also
has anti-hypertensive property

B If convulsion does not


recur then pregnancy can be
continued with MgSO4 cover.

C Alpha methyldopa is drug


of choice for hypertension

D Ergometrine should be
avoided since it is a
vasoconstrictor

23.53% People got this right

Explanation:

Correct Answer (D)

Ergometrine should be avoided since it is a vasoconstrictor

MgSO4 is an anti-convulsant drug of choice for eclampsia, it acts on the NMDA


receptors in central nervous system. It increases the blood flow to CNS but it is
not an anti hypertensive drug. Eclampsia requires termination of pregnancy after
stabilisation irrespective of the period of gestation. There is no scope of
continuing pregnancy. The drug of choice to control hypertension in eclampsia

https://emedicoz.com/testresult/683510s14692547 138/189
6/9/24, 12:45 PM Neet PG Preparation, Neet PG Coaching, FMGE, USMLE

is Labetolol. Post- Delivery of the baby, Ergot derivatives are avoided since these
drugs increase the hypertension due to vaso constrictive properties.

(Reference: Williams obstetrics, Obstetric complications, Hypertension in


pregnancy, Eclampsia)

Question: 149
Which of the following is incorrect regarding thyroid disorder in
pregnancy?

A Most common cause of


thyrotoxicosis is Grave’s disease

B Most common cause of


hypothyroidism is iodine
deficiency

C Propylthiouracil is the drug


of choice in first trimester for
hyperthyroidism

D Methimazole can be given


from second trimester for
hyperthyroidism

31.68% People got this right

Explanation:

Correct Answer (B)

The most common cause of Hypothyroidism is Hashimoto’s thyroiditis which is


due to Anti-TPO antibodies. Hypothyroidism can lead to abortion. There is an
increase in the demand of iodine supplementation in pregnancy. Other
statements are correct in the given question.

https://emedicoz.com/testresult/683510s14692547 139/189
6/9/24, 12:45 PM Neet PG Preparation, Neet PG Coaching, FMGE, USMLE

(Reference: Williams obstetrics, Medical disorders, endocrinological disorders,


hypothyroidism.)

Question: 150
Primigravida with 22 weeks of gestation with Hb 7 gm%, not able to
tolerate oral iron therapy is subjected to parenteral iron therapy (iron
sucrose). Her weight is 50 kg. Calculate the amount of iron to be given to
this patient:

A 960mg

B 980mg

C 1020mg

D 860mg

11.41% People got this right

Explanation:

Correct Answer (B)

980 mg

To calculate the dose of injectable iron, we use Ganzoni formula, it is (2.4 x


weight in kg x Hb deficit) + 500 mg for iron stores. In this case weight is 50kg
and her Hb is 7 gm%. We need to reach the target of 11gm% so the Hb deficit is
4gm%. It comes out as 480+500mg = 980mg

(Reference: Anemia Mukt Bharat guidelines)

https://emedicoz.com/testresult/683510s14692547 140/189
6/9/24, 12:45 PM Neet PG Preparation, Neet PG Coaching, FMGE, USMLE

Question: 151
Which one of the following is not a WHO category IV heart disease
patient?

A CLASS 3 NYHA

B LVEF 25%

C Heart transplantation

D Pulmonary arterial
hypertension

20.93% People got this right

Explanation:

Correct Answer (C)

Heart transplantation

Patient with past history of heart transplantation falls under category III of WHO.
Category IV of WHO will consist of Class 3, class 4 NYHA, LVEF <30%, severe left
ventricular obstruction, pulmonary arterial hypertension, Marfan’s syndrome
with aortic root dilatation of >4cm and peripartum cardiomyopathy with residual
ventricular dysfunction. In all such cases, continuation of pregnancy is
contraindicated.

(Reference: Williams obstetrics, Medical disorders, CVS disorders, WHO category


Table.)

Question: 152
Primigravida with 24 weeks of gestation has come for the first time for
ante natal visit. Her DIPSI test was 168mg%, HbA1c was 7.8%. Usg Obs

https://emedicoz.com/testresult/683510s14692547 141/189
6/9/24, 12:45 PM Neet PG Preparation, Neet PG Coaching, FMGE, USMLE

was within normal limits. TIFFA scan was within normal limits. Which of
the following is not true?

A The drug of choice is


insulin therapy

B She is a case of pre


gestational DM

C Ideal blood sugar levels are


F <95, 1hr<140, 2hr<120mg%

D IUGR cannot occur in this


case

24.77% People got this right

Explanation:

Correct Answer (D)

IUGR cannot occur in this case

This is a patient of overt Dm or pre-gestational diabetes mellitus since HbA1c is


higher than 6.5% which means the last 3 months had abnormal blood sugar
levels. She has come to us at 24 weeks of gestation. Insulin is must in the
treatment of overt DM. Oral hypoglycemic agents are not given in overt DM.
according to American Diabetes Association, the target blood suger levels which
must be maintained are F<95, 1Hr <140 and 2Hr < 120mg%. in overt DM there
is a possibility of IUGR due to vasculopathy.

(Reference: Williams obstetrics, Medical disorders, Diabetes Mellitus, Overt DM)

Question: 153
Which one of the following patients is not suitable for metformin
therapy?

https://emedicoz.com/testresult/683510s14692547 142/189
6/9/24, 12:45 PM Neet PG Preparation, Neet PG Coaching, FMGE, USMLE

A GDM with fasting value


110mg%

B Overt DM

C HAIR-AN syndrome

D PCOS with raised fasting


insulin levels

26.37% People got this right

Explanation:

Correct Answer (B)

Overt DM

Overt Dm cases are to be treated by insulin therapy. Oral hypoglycemic therapy


is not suitable in these cases. In PCOS patients with Insulin resistance, Metformin
is very effective in maintaining hormonal balance and hence it helps in ovulation
in PCOS.

(Reference: Williams obstetrics, Medical disorders, Diabetes mellitus, Overt DM)

Question: 154
One of the following is not a bad prognostic marker in Gestational
Trophoblastic neoplasia?

https://emedicoz.com/testresult/683510s14692547 143/189
6/9/24, 12:45 PM Neet PG Preparation, Neet PG Coaching, FMGE, USMLE

A Previous pregnancy = full


term delivery

B b-HCG >100000IU/L

C Lung metastasis

D Interval from index


pregnancy > 12 months.

28.31% People got this right

Explanation:

Correct Answer (C)

Lung metastasis

Lung and pelvic metastasis are considered as good prognostic markers in GTN.
Most common site of metastasis is lungs. Poor prognostic markers are: Age>40,
B-HCG> 1 lac/cc, previous pregnancy as full term deliveries, brain, liver, GI
metastasis, interval between pregnancy and cancer >1 year, previous failed
chemotherapy and number and size of metastasis.

(Reference: WHO’s prognostic marker table of GTN and William’s Obstetrics,


gestational trophoblastic diseases)

Question: 155
A 23 years old G2P1L1 with 13 weeks of gestation has a history of passage
of grape like vesicles for 1 day. On examination there is snow-storm
appearance of products of conception. She is subjected to suction and
evacuation. Her baseline b-HCG is 85000IU/L. Which of the following is
correct regarding the further management?

https://emedicoz.com/testresult/683510s14692547 144/189
6/9/24, 12:45 PM Neet PG Preparation, Neet PG Coaching, FMGE, USMLE

A She should be subjected to


weekly b-HCG till it is
undetectable and then monthly
b-HCG for 12 months, COC pills
for contraception.

B She should be subjected to


weekly b-HCG till it is
undetectable and then monthly
b-HCG for 6 months, Cu-T for
contraception

C She should be subjected to


weekly b-HCG till it is
undetectable and then monthly
b-HCG for 6 months, COC pills for
contraception

D Weekly b-HCG till it is


undetectable. Once it is negative,
she is allowed to conceive if she
desires.

39.64% People got this right

Explanation:

Correct Answer (C)

She should be subjected to weekly b-HCG till it is undetectable and then


monthly b-HCG for 6 months, COC pills for contraception

This is a case of complete molar pregnancy which has 15-20 % risk of malignant
transformation. Treatment of choice is suction and evacuation followed by HCG
monitoring for 6 months after it is undetectable for 3 times. In this time period,
patient is advised to use contraceptive for 6 months. Copper T is never used
since there is a chance of bleeding in these cases so copper-T is contraindicated.
COC pills are best in these cases

https://emedicoz.com/testresult/683510s14692547 145/189
6/9/24, 12:45 PM Neet PG Preparation, Neet PG Coaching, FMGE, USMLE

(Reference: William’s obstetrics, 25th edition, Gestational Trophoblastic Disease,


Molar pregnancy)

Question: 156
A 32 years old nulliparous patient had history of 3 abortions in last 3
years. All her abortions were spontaneous and were of 8-9 weeks. What is
the possible cause amongst the following?

A Cervical incompetence

B Syphilis

C Balanced translocation in
patient and partner

D Subserous pedunculated
fibroids

37.52% People got this right

Explanation:

Correct Answer (C)

Balanced translocation in patient and partner

Genetic cause is a rare cause of recurrent pregnancy losses in which there are
balanced translocation in the female or male partner. Cervical incompetence can
never cause first trimester abortions. Syphilis never affects 1st trimester since it
requires larges vessels to pass through placenta. Subserous fibroids do not
distort uterine cavity so there is no chance of abortion in these cases.

(Reference: William’s obstetrics, 25th edition, Abortions, recurrent abortions)

https://emedicoz.com/testresult/683510s14692547 146/189
6/9/24, 12:45 PM Neet PG Preparation, Neet PG Coaching, FMGE, USMLE

Question: 157
A 25 years old primigravida with tubal ectopic pregnancy of 6 weeks was
subjected to single dose therapy (medical management). Her b-HCG
value is 2500 IU/L on day 4 of treatment. After three days of
methotrexate injection, her b-HCG is 3250 IU/L. What is the next best line
of management?

A Give inj. Mtx again

B Repeat the b-HCG test


after 3 days

C Laparoscopic
salpingostomy

D pgf2a injection

22.16% People got this right

Explanation:

Correct Answer (A)

Give inj MTx again

In a case of single dose methotrexate therapy, D1 Inj. MTX is given, D4 HCG is


checked, D7 HCG is checked again. If this value is 15% less than D4 value then
the treatment is considered as over. In this case, it has not decreased by 15 % on
day 7 so we need to repeat the course again.

(Reference: William’s obstetrics, 25th edition, ectopic pregnancy, medical


management)

Question: 158
https://emedicoz.com/testresult/683510s14692547 147/189
6/9/24, 12:45 PM Neet PG Preparation, Neet PG Coaching, FMGE, USMLE

According to Gainsville’s classification, if there is acute salpingitis with


peritonitis then it is considered as?

A Stage 1

B Stage 2

C Stage 3

D Stage 4

9.36% People got this right

Explanation:

Correct Answer (B)

Stage 2

Gainesville staging of Acute PID:

Stage I: Acute endometritis-salpingitis without peritonitis

Stage II: Acute salpingitis with peritonitis

Stage III: Acute salpingitis with superimposed tubal occlusion

Stage IV: Ruptured tubo-ovarian abscess or tubo-ovarian complex

Stage V: Respiratory complications

Question: 159
All are true regarding bacterial vaginosis except?

A Condom use reduces the


risk

https://emedicoz.com/testresult/683510s14692547 148/189
6/9/24, 12:45 PM Neet PG Preparation, Neet PG Coaching, FMGE, USMLE

B Patient complains of
vaginal discharge with itching

C It is not a sexually
transmitted infection

D It is more common in
patients with presence of sexually
transmitted infection

24.51% People got this right

Explanation:

Correct Answer (B)

Patient complains of vaginal discharge with itching

Bacterial vaginosis is most common vaginal infection. It is change in normal


floral environment of vagina. Lactobacilli are reduced. pH increases >4.5. most
common organism is Gardnerella vaginalis. There is no inflammation so there is
no itching or redness. Patient complains of fishy foul smell discharge. Though it
is not a sexually transmitted disease, patients with STI have altered flora of
vagina which leads to bacterial vaginosis. Barrier contraceptives prevent STI so
there is prevention of Bacterial Vaginosis as well.

(Reference: Dutta’s Gynecology, vaginal infection)

Question: 160
A 32 years old sexually active nulliparous patient has come with history of
burning micturition with vaginal discharge since last 7 days. On
examination, there are white flakes seen on vaginal epithelium. On
removal of those flakes, there is severe inflammation seen in vagina. Her
partner is not having any such symptom. What would be most likely
found on wet mount examination?

https://emedicoz.com/testresult/683510s14692547 149/189
6/9/24, 12:45 PM Neet PG Preparation, Neet PG Coaching, FMGE, USMLE

A Vaginal epithelial cells with


irregular margins

B Spores and rods forming


pseudo-hyphae

C Unicellular motile protozoa

D Vaginal epithelial cells with


inclusion bodies

40.95% People got this right

Explanation:

Correct Answer (B)

Spores and rods forming pseudo-hyphae

This is a case of candidiasis. Wet mount shows spores and filaments forming
pseudo hyphae.

(Reference: Dutta’s Gynecology, vaginal infection)

Question: 161
One of the following is included in specific criteria for diagnosis of PID as
per CDC guideline?

A C. Trachomatis or
N.Gonorrhoea NAAT positive

B Endometrial biopsy
suggestive of endometritis

C Mucopurulent vaginal

https://emedicoz.com/testresult/683510s14692547 150/189
6/9/24, 12:45 PM Neet PG Preparation, Neet PG Coaching, FMGE, USMLE

discharge

D Cervical motion tenderness

15.64% People got this right

Explanation:

Correct Answer (B)

Endometrial biopsy suggestive of endometritis

(Ref: William’s gynecology, genital infections, PID)

Question: 162
One of the following is not correct regarding boundaries of ovarian fossa:

A Superiorly by psoas major


muscle

B Anteriorly and inferiorly by


broad ligament

C Posteriorly by internal iliac


vessels and ureter

https://emedicoz.com/testresult/683510s14692547 151/189
6/9/24, 12:45 PM Neet PG Preparation, Neet PG Coaching, FMGE, USMLE

D Inferiorly by obturator
vessels and nerve

20.67% People got this right

Explanation:

Correct Answer (A)

Superiorly by psoas major muscle

Question: 163
Which of the following is not true regarding NEXPLANON contraceptive?

A It contains etonogestrel

B It is radio opaque device

C It is safe in DVT since it is


progesteronic

D It must be removed after 3


years

14.15% People got this right

https://emedicoz.com/testresult/683510s14692547 152/189
6/9/24, 12:45 PM Neet PG Preparation, Neet PG Coaching, FMGE, USMLE

Explanation:

Correct Answer (C)

It is safe in DVT since it is progesteronic

Though it is progesteronic, it has increased chance of thrombosis especially in


smokers.

(Ref: www.Nexplanon.com)

Question: 164
Which of the following is not true regarding this contraceptive?

A Endometrial atrophy
prevents implantation

B Progesterone inhibits
ovulation by inhibiting LH surge

https://emedicoz.com/testresult/683510s14692547 153/189
6/9/24, 12:45 PM Neet PG Preparation, Neet PG Coaching, FMGE, USMLE

C Also used in treatment of


endometrial hyperplasia without
atypia

D Safe in patients with


estrogenic side effects

30.12% People got this right

Explanation:

Correct Answer (B)

Progesterone inhibits ovulation by inhibiting LH surge

It is Mirena, it contains LNG and it released LNG 20 mcg per day due to this
there is endometrial atrophy. The amount of progesterone released is not
sufficient to inhibit LH levels so it does not affect ovulation. It is given in
premalignancy of uterus without atypia.

(Ref: speroff gynecology, contraception, IUD)

Question: 165
According to MTP act (amendment 2020, passed in 2021 which came into
effect from 24th September 2021), MTP for pregnancy due to rape is
allowed till?

A 20 weeks and 1 doctor is


needed

B 24 weeks and 2 doctors are


needed for 20-24 week MTP

C 24 weeks and 1 doctor is

https://emedicoz.com/testresult/683510s14692547 154/189
6/9/24, 12:45 PM Neet PG Preparation, Neet PG Coaching, FMGE, USMLE

needed

D 20 weeks and 2 doctors are


needed

54.17% People got this right

Explanation:

Correct Answer (B)

24 weeks and 2 doctors are needed for 20-24 weeks MTP

(Ref: THE GAZETTE OF INDIA, MTP ACT AMENDMENT)

Question: 166
Identify the following instrument:

A Veress needle

B Uterine manipulator

C HSG cannula

D Myoma screw

https://emedicoz.com/testresult/683510s14692547 155/189
6/9/24, 12:45 PM Neet PG Preparation, Neet PG Coaching, FMGE, USMLE

44.25% People got this right

Explanation:

Correct Answer (C)

HSG cannula

This is HSG cannula also called as LEECH WILKINSON cannula. It is used to inject
contrast for hysterosalpingography. Water soluble iodine-based contrast is
routinely used.

Question: 167
Which tumour markers constitute “roma score” for ovarian malignancy?

A ca-125 and human


epidermal protein 4

B ca-125 and human


epididymis protein 4

C ca-125 and
carcinoembryonic antigen

D ca-125 and
chorioembryonic antigen

4.75% People got this right

Explanation:

Correct Answer (B)

ROMA criteria is RISK OF OVARIAN MALIGNANCY ASSESSMENT. It involves


assessment of CA-125 and Human Epididymis protein-4. Higher values would
indicate malignancy for which we need to do staging exploratory laparotomy.

(Ref: NOVAK gynecology, ovarian malignancy)

https://emedicoz.com/testresult/683510s14692547 156/189
6/9/24, 12:45 PM Neet PG Preparation, Neet PG Coaching, FMGE, USMLE

Question: 168
Which of the following is epithelial ovarian tumour?

A Yolk sac tumour

B Granulosa cell tumour

C Brenner tumour

D None of the above

38.08% People got this right

Explanation:

Correct Answer (C)

Brenner tumour

Epithelial ovarian cancers are most common ovarian cancers. Most common
type is serous cystadenocarcinoma. Other epithelial types are mucinous,
endometrioid, clear cell, Brenner and undifferentiated types.

(Ref: NOVAK gynecology, ovarian malignancy)

Question: 169
Most common site of metastasis for vulval cancer is?

A Cervix

https://emedicoz.com/testresult/683510s14692547 157/189
6/9/24, 12:45 PM Neet PG Preparation, Neet PG Coaching, FMGE, USMLE

B Obturator lymph nodes

C Inguinofemoral lymph
nodes

D Lungs

30.19% People got this right

Explanation:

Correct Answer (C)

Inguinofemoral lymph nodes

The lymphatic drainage of vulva (external genitalia) is into inguinofemoral lymph


nodes. Vulval cancers spread by lymphatics to inguinofemoral lymph nodes.

(Ref: NOVAK gynecology, vulval malignancy)

Question: 170
In a 25 years old nulliparous patient with amenorrhoea, the test which
can differentiate ovarian failure from endometrial failure is?

A Progesterone challenge
test

B Pulsatile GnRH stimulation


test

C Sr. FSH test

D Hysteroscopy

25.7% People got this right


https://emedicoz.com/testresult/683510s14692547 158/189
6/9/24, 12:45 PM Neet PG Preparation, Neet PG Coaching, FMGE, USMLE

Explanation:

Correct Answer (C)

Sr. FSH test

Sr. FSH is high in a case of ovarian failure. It is >/= 40 IU/L. ovarian failure is
hypergonadotropic hypogonadism. Sr. FSH will be normal in case of endometrial
failure e.g Ashermann syndrome. It will be 2-10 IU/L

(Ref: Speroff Gynecology, Amenorrhoea, secondary amenorrhoea and


evaluation)

Question: 171
Which Le fort fracture causes Cranio-facial disjonction?

A Type I & II

B Type II & III

C Type I and Type III

D Type III only

38.71% People got this right

Explanation:

Correct Answer (D)

https://emedicoz.com/testresult/683510s14692547 159/189
6/9/24, 12:45 PM Neet PG Preparation, Neet PG Coaching, FMGE, USMLE

• Le Fort type I

o horizontal maxillary fracture, separating the teeth from the upper face

o fracture line passes through the alveolar ridge, lateral nose and inferior wall
of the maxillary sinus

o also known as a Guerin fracture

• Le Fort type II

o pyramidal fracture, with the teeth at the pyramid base, and nasofrontal
suture at its apex

o fracture arch passes through the posterior alveolar ridge, lateral walls of
maxillary sinuses, inferior orbital rim and nasal bones

o uppermost fracture line can pass through the nasofrontal junction or the
frontal process of the maxilla 3

• Le Fort type III

o craniofacial disjunction

o transverse fracture line passes through nasofrontal suture, maxillo-frontal


suture, orbital wall, and zygomatic arch/zygomaticofrontal suture

o because of the involvement of the zygomatic arch, there is a risk of the


temporalis muscle impingement

o unsurprisingly type III fractures have the highest rate of CSF leak

Point to remember:

https://emedicoz.com/testresult/683510s14692547 160/189
6/9/24, 12:45 PM Neet PG Preparation, Neet PG Coaching, FMGE, USMLE

• Le Fort I is a floating palate (horizontal)

• Le Fort II is a floating maxilla (pyramidal)

• Le Fort III is a floating face (transverse)

Question: 172
Which of the following breast implant is associated with higher risk of
Anaplastic Large cell lymphoma?

A Textured implant

B Smooth Implant

C Subpetoral implant

D Sub glandular implants

7.82% People got this right

Explanation:

Correct Answer (A)

Breast implant–associated anaplastic large cell lymphoma (BIA-ALCL) is a new


provisional category in the 2016 World Health Organization (WHO) classification
of lymphoid neoplasms, median time of presentation is 10 years after implant
insertion.

Main presentation is breast implant effusion or mass

Etiological hypothesis is it is due to activation of T cells reacting to bacterial


biofilm arising in breast implants

The disease shares similar immunopathologic characteristics with systemic and


cutaneous anaplastic lymphoma kinase (ALK)–negative anaplastic large cell

https://emedicoz.com/testresult/683510s14692547 161/189
6/9/24, 12:45 PM Neet PG Preparation, Neet PG Coaching, FMGE, USMLE

lymphoma (ALCL), with densely positive CD30 expression and negative ALK
expression.

The majority of patients with the peri-implant effusion subtype have indolent
disease and a good outcome, as opposed to the poorer-prognosis mass-
forming or advanced-stage subtype. Unlike with other lymphomas, surgical
excision is recommended as first-line (and curative) therapy in most cases of
BIA-ALCL, with systemic therapy reserved for mass-forming disease, distant
disease, and relapsed or refractory disease.

Question: 173
According to Jackson burn model, Non salvageable zone in burn is?

A Zone of coagulation

B Zone of stasis

C Zone of Hyperaemia

D All zone are salvageable

43.92% People got this right

Explanation:

Correct Answer (A)

https://emedicoz.com/testresult/683510s14692547 162/189
6/9/24, 12:45 PM Neet PG Preparation, Neet PG Coaching, FMGE, USMLE

Question: 174
Identify the instrument used for facial bone fixation?

A Erich arch bars

B Bridle wire

C Plate fixation

D Compression screws

12.5% People got this right

Explanation:

Correct Answer (C)

https://emedicoz.com/testresult/683510s14692547 163/189
6/9/24, 12:45 PM Neet PG Preparation, Neet PG Coaching, FMGE, USMLE

Question: 175
In laparoscopic hernia surgery, corona mortis refers to?

A An Ectopic gonadal vessel


found in triangle of doom

B An aberrant artery between


obturator and the external iliac or
inferior epigastric arteries

C Deep inferior epigastric


artery

D Higher origin of profunda


femoris

46.72% People got this right

Explanation:

Correct Answer (B)

https://emedicoz.com/testresult/683510s14692547 164/189
6/9/24, 12:45 PM Neet PG Preparation, Neet PG Coaching, FMGE, USMLE

Question: 176
Diversion colitis seen after end colostomy is due to deficiency of?

A SCFA

B Glutamate

C Triglycerides

D Essential fatty acids

26.51% People got this right

Explanation:

Correct Answer (A)

Diversion colitis

• Proximal fecal diversion by ileostomy or colostomy leads to distal Mucosal


atrophy.

• Colonic mucosa derives energy by

• Short chain fatty acids (SCFA) produced by

• Bacterial fermentation of dietary

https://emedicoz.com/testresult/683510s14692547 165/189
6/9/24, 12:45 PM Neet PG Preparation, Neet PG Coaching, FMGE, USMLE

• Carbohydrate.

• Milder forms are self-limiting

• Treatment is short-chain fatty acid irrigation, steroid enemas and mesalazine

Question: 177
A 48-year-old woman develops pain in the right lower quadrant while
playing tennis. The pain progresses and the patient presents to the
emergency room later that day with a low-grade fever, Hg 11 gm% and
WBC count of 10,000/mm3 and complaints of anorexia and nausea as well
as persistent, sharp pain in lower abdomen (more on right side). On
examination, she is tender in the right lower quadrant with muscular spasm,
and Carnett’s sign and Fothergill’s sign were positive. An ultrasound is
ordered and shows an apparent mass in the abdominal wall. This condition
is associated with all except?

A Pregnancy.

B Strenuous muscular
exercise.

C Anti coagulant.

https://emedicoz.com/testresult/683510s14692547 166/189
6/9/24, 12:45 PM Neet PG Preparation, Neet PG Coaching, FMGE, USMLE

D Benign and malignant


tumour.

10.61% People got this right

Explanation:

Correct Answer (D)

This is rectus sheath hematoma-

A rectus sheath hematoma is an accumulation of blood in the sheath of the


rectus abdominis muscle. It causes abdominal pain with or without a mass.

The hematoma may be caused by either rupture of the epigastric artery or by a


muscular tear. Causes are anticoagulation, sudden straining (coughing),
pregnancy, abdominal surgery and trauma.

Generally managed conservatively

Question: 178
Which of the following is not a surgery for Pilonidal sinus?

A Bascom operation

B Karydakis operation

C Limberg Flap

D Karapandzic flap

21.74% People got this right

Explanation:

Correct Answer (D)

https://emedicoz.com/testresult/683510s14692547 167/189
6/9/24, 12:45 PM Neet PG Preparation, Neet PG Coaching, FMGE, USMLE

(Karapandzic flap is done for lip reconstruction)

1. Surgery for Pilonidal sinus Unroofing the tract, curetting the base, and
marsupializing the wound.

2. Simple cystectomy with primary closure.

3. Other methods (for complex and/or recurrent sinus tracts)-

Extensive resection and closure with

• Karydakis flap

• Bascom procedure

• Limberg flap

• Z-plasty

• Y-V plasty

• Myofascial flap closure

Question: 179
Which of the following is not a radiological sign of Sigmoid volvulus

A Steelpan sign

B King of spade sign

C Northern exposure sign

D Coiled spring sign

25.65% People got this right

Explanation:

Correct Answer (D)

https://emedicoz.com/testresult/683510s14692547 168/189
6/9/24, 12:45 PM Neet PG Preparation, Neet PG Coaching, FMGE, USMLE

Coiled spring sign is seen in Bowel hematoma or rarely in Intussusception

Question: 180
Piggyback technique is used for?

A Liver transplant

B Pancreatic transplant

C Intestinal transplant

D Islet cell transplant

28.56% People got this right

Explanation:

Correct Answer (A)

The piggyback technique is used in orthotopic liver transplantation to simplify


the liver transplant. Rather than procuring the anastomosing IVC to IVC which
will need Veno venous bypass VB in this technique donor IVC directly to the
hepatic veins of the recipient.

https://emedicoz.com/testresult/683510s14692547 169/189
6/9/24, 12:45 PM Neet PG Preparation, Neet PG Coaching, FMGE, USMLE

Question: 181
Which of the following type of abscess can become horse-shoe abscess?

A Peri anal abscess

B Inter-sphincteric abscess

C Ischio-rectal abscess

D Supra-levator abscess

32.82% People got this right

Explanation:

Correct Answer (C)

https://emedicoz.com/testresult/683510s14692547 170/189
6/9/24, 12:45 PM Neet PG Preparation, Neet PG Coaching, FMGE, USMLE

Question: 182
Established recommended treatment for Pseudomyxoma peritonei is?

A Cisplatin based
chemotherapy

B Ipilimumab (CTLA-4
inhibitor)

C Whole abdominopelvic
radiotherapy (WAPRT)

D CRS and HIPEC

25.74% People got this right

Explanation:

Correct Answer (D)

Cytoreductive surgery (CRS) and hyperthermic intraperitoneal chemotherapy


(HIPEC) is indicated in primary (Mesothelioma) or secondary malignancies of the
peritoneum (e.g. colorectal cancer, appendix neoplasms, pseudomyxoma
peritonei, ovarian cancer).

After a cytoreductive surgery intraperitoneal chemotherapy is instilled at 42


degree C for 2- 6 hours
https://emedicoz.com/testresult/683510s14692547 171/189
6/9/24, 12:45 PM Neet PG Preparation, Neet PG Coaching, FMGE, USMLE

Question: 183
Which of the following is not recommended in enhanced recovery
programme for colorectal cancer surgery

A Thorough and extensive


mechanical bowel preparation

B Preoperative carbohydrate
loading

C No nasogastric tubes

D Avoidance of perioperative
fluid/salt overload

8.61% People got this right

Explanation:

Correct Answer (A)

Question: 184

https://emedicoz.com/testresult/683510s14692547 172/189
6/9/24, 12:45 PM Neet PG Preparation, Neet PG Coaching, FMGE, USMLE

Best investigation to detect prostate secondaries is?

A 18 FDG PET scan

B 68 Ga PSMA PET CT

C CECT

D <p>Dotatate
Inidium<sup>111</sup>
SRS</p>

25.28% People got this right

Explanation:

Correct Answer (B)

The sensitivity and specificity of PSA values is not very high due to lot of inter
and intra-individual variations. The significance of PSA values has to be
correlated with imaging modalities since lymph node and skeletal metastases
can be present without any obvious increase in PSA values or PSA levels could
increase after chemotherapy without any obvious underlying disease.

Prostate specific membrane antigen (PSMA is a peptidase and a more robust


surface biomarkers. Prostate-specific membrane antigen (PSMA) is a cell surface
protein expressed abundantly in Prostate Cancer cells.

Studies have shown a strong correlation between PSMA, stage of disease and
Gleason's score. Hormone refractory cancers also show high PSMA and high
PSMA is an independent marker of disease recurrence.

https://emedicoz.com/testresult/683510s14692547 173/189
6/9/24, 12:45 PM Neet PG Preparation, Neet PG Coaching, FMGE, USMLE

The yellow arrow points to an area with an overexpression of the antigens


(recurrence).

Question: 185
Which of the following is not an advantage of Meshing of split thickness
graft

A Increases surface area

B Helps in egress of
collection from below the graft

C Better cosmesis

D Increases flexibility of graft

38.45% People got this right

Explanation:

Correct Answer (C)

It gives a poor cosmesis (Crocodile skin)

https://emedicoz.com/testresult/683510s14692547 174/189
6/9/24, 12:45 PM Neet PG Preparation, Neet PG Coaching, FMGE, USMLE

Question: 186
Most common site of obstruction in Hydrocephalus is?

A Foramen of Munro

B Acquaduct of Silvius

C Foramen of Luschka and


Magendi

D Sub choroid plexus

10.61% People got this right

Explanation:

Correct Answer (A)

https://emedicoz.com/testresult/683510s14692547 175/189
6/9/24, 12:45 PM Neet PG Preparation, Neet PG Coaching, FMGE, USMLE

Question: 187
If TURP is extended below Veru Montanum, there is higher risk of?

A Infection

B Hemorrhage

C Incontinence

D Capsular perforation

51.26% People got this right

Explanation:

Correct Answer (C)

External sphincter is just below Veru, so any excision below veru makes it
susceptible for injury to sphincter, leading to incontinence

https://emedicoz.com/testresult/683510s14692547 176/189
6/9/24, 12:45 PM Neet PG Preparation, Neet PG Coaching, FMGE, USMLE

Question: 188
A patient presented with a thyroid nodule with euthyroid status. FNAC
reported Bathesda score 5. What is the ideal advise for this patient?

A Close observation

B Repeat FNAC as it is
inconclusive

C Genetic counselling (BRCA,


RET-PTC)

D Surgery

39.11% People got this right

Explanation:

Correct Answer (D)

https://emedicoz.com/testresult/683510s14692547 177/189
6/9/24, 12:45 PM Neet PG Preparation, Neet PG Coaching, FMGE, USMLE

Question: 189
False statement about parathyroid is?

A Superior parathyroid is
deeper to RLN while inferior is
superficial to RLN

B PTH increases calcium


absorption from Kidney so
hyperparathyroidism is
characterized by decreased
urinary calcium

C Most common cause of


Primary hyperparathyroidism is
solitary adenoma

D In De George syndrome
there is absence of parathyroid
and Thymus.

14.48% People got this right

Explanation:

Correct Answer (B)

https://emedicoz.com/testresult/683510s14692547 178/189
6/9/24, 12:45 PM Neet PG Preparation, Neet PG Coaching, FMGE, USMLE

In hyperparathyroidism there is hypercalciuria due to very high serum calcium


levels. Although there is increase in calcium absorption by kidney but the level in
blood is much higher than threshold value for renal absorption.

Question: 190
Which of the following finding has been removed from 10th addition of
ATLS guidelines as suggestive sign of pelvic fracture (Although it was
included in 9th edition).

A High riding prostate in PR


examination

B Scrotal haematoma and


blood at urethral meatus

C Discrepancy in limb length

D Rotational deformity of leg


without obvious fracture

15.11% People got this right

Explanation:

Correct Answer (A)

https://emedicoz.com/testresult/683510s14692547 179/189
6/9/24, 12:45 PM Neet PG Preparation, Neet PG Coaching, FMGE, USMLE

Question: 191
False statement about Diaphragmatic injury is?

A Left side is more common


than right

B Laparoscopy is nest
diagnostic modality.

C Delayed presentation in
common (due to herniation)

D Repair is best done from


thoracotomy

26.79% People got this right

Explanation:

Correct Answer (D)

Repair is best done by Laparotomy

Question: 192

https://emedicoz.com/testresult/683510s14692547 180/189
6/9/24, 12:45 PM Neet PG Preparation, Neet PG Coaching, FMGE, USMLE

Which type of dressing has highest absorptive capacity?

A Hydrogel dressing

B Hydrocolloid dressing

C Calcium Alginate

D Silicone dressings

4.73% People got this right

Explanation:

Correct Answer (D)

Silicone dressing is used as compression dressing for hypertrophic scar

Hydrogel Dressings

• Wounds that are dry or mostly dry; any wound with dead tissue

• Lend moisture to a wound and help breakdown dry and dead tissue.

• Promote wound healing – Burns

Hydrocolloid Dressings

• For lightly to moderate exuding wounds and non-viable tissue

Calcium Alginates

• Wounds with moderate to heavy discharge hold as much as 20 times its


weight in moisture

Question: 193
Flow rate through a pink IV cannula is?

https://emedicoz.com/testresult/683510s14692547 181/189
6/9/24, 12:45 PM Neet PG Preparation, Neet PG Coaching, FMGE, USMLE

A 20 ml/min

B 30 ml/ min

C 60 mi/min

D 90 ml/min

33.31% People got this right

Explanation:

Correct Answer (C)

Pink cannula is 20G which has a flow rate of 60 ml/ min

Question: 194
Small bowel obstruction after Roux en Y Gastric bypasss should be
treated as an urgent surgical emergency because?

A It is frequently due to an
incarcerated internal hernia which
can progress to bowel necrosis
and perforation

B Abdominal distension risks


disruption of suture lines

C Signs and symptoms of


peritonitis, such as pain, fever,
and leukocytosis, are usually
masked in the obese

D Nasogastric intubation will


not decompress the distal gastric

https://emedicoz.com/testresult/683510s14692547 182/189
6/9/24, 12:45 PM Neet PG Preparation, Neet PG Coaching, FMGE, USMLE

remnant

35.89% People got this right

Explanation:

Correct Answer (A)

Petersen Hernia is a common complication of RYGB

Small bowel obstruction after RYGB is frequently due to an incarcerated internal


between the mesentery of Roux limb and mesocolon (or through other
iatrogenic mesentery defects). This can progress rapidly to strangulation and
necrosis of the bowel with subsequent perforation. Adverse outcomes with this
complication have resulted in the uniform recommendation that small bowel
obstruction in this setting should be regarded as a surgical emergency.
Abdominal distention and difficulties with nasogastric intubation are not
relevant concerns

Question: 195
Sequence of anastomosis in hand reimplantation is?

A Bone fixation > Nerve


repair > artery repair > Venous
anastomoses > flexor tendon >
extensor tendon repair > skin +/-
fasciotomy

https://emedicoz.com/testresult/683510s14692547 183/189
6/9/24, 12:45 PM Neet PG Preparation, Neet PG Coaching, FMGE, USMLE

B Bone fixation > extensor


tendon repair > Flexor tendon
repair > Nerve repair > Venous
anastomoses > skin +/-
fasciotomy

C Bone fixation > Flexor


tendon > Nerve > artery repair >
Venous anastomoses > Extensor
repair > skin +/- fasciotomy

D Bone fixation > extensor


tendon repair > artery repair >
Venous anastomoses > flexor
tendon > nerve repair > skin +/-
fasciotomy

11.5% People got this right

Explanation:

Correct Answer (D)

Warm Ischemia time – for hand 12 hrs, For arm/ forearm 24 hrs

Cold ischemia time- For hand 6 hours and for arm/ forearm 12 hrs

Question: 196
A 32 year old man is presented with the following clinical presentation

What is the most probable diagnosis?

https://emedicoz.com/testresult/683510s14692547 184/189
6/9/24, 12:45 PM Neet PG Preparation, Neet PG Coaching, FMGE, USMLE

A Poland's Anomaly

B Cleidocranial dysostosis

C Congenital Muscular
torticollis

D Klippel Fiel Syndrome

44.53% People got this right

Explanation:

Correct Answer (A)

Absence of ipsilateral pectoralis major + Synbrachydactyly + Sprengel's shoulder


= Poland's Anomaly

Question: 197
A 9 year old child is brought by his parents with chief complains of pain
and swelling in right thigh which have exaggerated since last couple of
https://emedicoz.com/testresult/683510s14692547 185/189
6/9/24, 12:45 PM Neet PG Preparation, Neet PG Coaching, FMGE, USMLE

weeks. His Thorough examination was done earlier which revealed


Increased TLC, Neutrophils and ESR. His repeated blood cultures went
Negative. X Ray Was done which showed lesion in femur mainly in
diaphysis with some laminated periosteal reaction. What is the NEXT
investigation that should be done?

A Bone Scan

B Biopsy

C MRI

D CT Scan

27.42% People got this right

Explanation:

Correct Answer (C)

Next step : MRI

Best step : Biopsy

Question: 198
A 42 year old female presents with tingling and numbness in right hand
that awakens her in the middle of the night for last two weeks. She also
gives history of being treated for diabetes and hypothyroidism for last 3
years by an endocrinologist. Which of the following ain't true about this
condition from the following options?

A Splintage is helpful only in


few initial months of onset of

https://emedicoz.com/testresult/683510s14692547 186/189
6/9/24, 12:45 PM Neet PG Preparation, Neet PG Coaching, FMGE, USMLE

symptoms

B Motor Symptoms appear


early in the course of the disease

C Durakn's Test is the most


specific clinical test in the
diagnosis

D It is the most common


focal compression neuropathy

40.76% People got this right

Explanation:

Correct Answer (B)

Motor Symptoms appear early in the course of the disease

Appearance of motor symptoms indicate advanced stages of the disease and


warrant immediate decompression surgery

Question: 199
An 8 year old boy presents with Right Hip Pain and Limping Gait for last
one week. The mothers gives history of recent URTI in last month only.
The examination revealed limitation of hip movements only in extremes.
Blood counts are almost normal except Raised ESR. What is the
diagnosis?

A Perthes' Disease

B Transient Synovitis

C Septic Arthritis

https://emedicoz.com/testresult/683510s14692547 187/189
6/9/24, 12:45 PM Neet PG Preparation, Neet PG Coaching, FMGE, USMLE

D TB hip

21.39% People got this right

Explanation:

Correct Answer (B)

Transient Synovitis is the most common cause of limping child where there is
recent history of URTI and almost normal hip movements.

Question: 200
Identify the test being performed in this image

A Finkelstein's Test

B Phalen's test

C Durkan's Test

D Adson's Test

50.02% People got this right

Explanation:

https://emedicoz.com/testresult/683510s14692547 188/189
6/9/24, 12:45 PM Neet PG Preparation, Neet PG Coaching, FMGE, USMLE

Correct Answer (A)

Finkelstein's test is a test used to diagnose de Quervain's tenosynovitis in people


who have wrist pain. Classical descriptions of the Finkelstein's test are when the
examiner grasps the thumb and ulnar deviates the hand sharply.

https://emedicoz.com/testresult/683510s14692547 189/189

You might also like